maternity 3

Réussis tes devoirs et examens dès maintenant avec Quizwiz!

12. Which finding would indicate to the nurse that a woman's cervix is ripe in preparation for labor induction? A) Posterior position B) Firm C) Closed D) Shortened

Ans: D Feedback: A ripe cervix is shortened, centered (anterior), softened, and partially dilated. An unripe cervix is long, closed, posterior, and firm.

14. The nurse is teaching a group of students about the similarities and differences between newborn skin and adult skin. Which statement by the group indicates that additional teaching is needed? A) The newborn's skin and that of an adult are similar in thickness. B) The lipid composition of the skin of a newborn and adult is about the same. C) Skin development in the newborn is complete at birth. D) The newborn has more fibrils connecting the dermis and epidermis.

Ans: C Feedback: The newborn and adult epidermis is similar in thickness and lipid composition, but skin development is not complete at birth. Fewer fibrils connect the dermis and epidermis in the newborn when compared with the adult.

14. Which of the following would alert the nurse to suspect that a newborn has developed NEC? A) Irritability B) Sunken abdomen C) Clay-colored stools D) Bilious vomiting

Ans: D Feedback: The newborn with NEC may exhibit bilious vomiting with lethargy, abdominal distention and tenderness, and bloody stools.

14. Which of the following would alert the nurse to suspect that a preterm newborn is in pain? A) Bradycardia B) Oxygen saturation level of 94% C) Decreased muscle tone D) Sudden high-pitched cry

Ans: D Feedback: The nurse should suspect pain if the newborn exhibits a sudden high-pitched cry, oxygen desaturation, tachycardia, and increased muscle tone.

23. The nurse is assessing a preterm newborn who is in the neonatal intensive care unit (NICU) for signs and symptoms of overstimulation. Which of the following would the nurse be least likely to assess? A) Increased respirations B) Flaying hands C) Periods of apnea D) Decreased heart rate

Ans: A Feedback: Conversely, overstimulation may have negative effects by reducing oxygenation and causing stress. A newborn reacts to stress by flaying the hands or bringing an arm up to cover the face. When overstimulated, such as by noise, lights, excessive handling, alarms, and procedures, and stressed, heart and respiratory rates decrease and periods of apnea or bradycardia may occur.

26. A pregnant woman is receiving misoprostol to ripen her cervix and induce labor. The nurse assesses the woman closely for which of the following? A) Uterine hyperstimulation B) Headache C) Blurred vision D) Hypotension

Ans: A Feedback: A major adverse effect of the obstetric use of Cytotec is hyperstimulation of the uterus, which may progress to uterine tetany with marked impairment of uteroplacental blood flow, uterine rupture (requiring surgical repair, hysterectomy, and/or salpingo-oophorectomy), or amniotic fluid embolism. Headache, blurred vision, and hypotension are associated with magnesium sulfate.

11. A woman is to undergo an amnioinfusion. Which statement would be most appropriate to include when teaching the woman about this procedure? A) "You'll need to stay in bed while you're having this procedure." B) "We'll give you an analgesic to help reduce the pain." C) "After the infusion, you'll be scheduled for a cesarean birth." D) "A suction cup is placed on your baby's head to help bring it out."

Ans: A Feedback: An amnioinfusion involves the instillation of a volume of warmed, sterile normal saline or Ringer's lactate into the uterus via an intrauterine pressure catheter. The client must remain in bed during the procedure. The use of analgesia is unrelated to this procedure. A cesarean birth is necessary only if the FHR does not improve after the amnioinfusion. Application of a suction cup to the head of the fetus refers to a vacuum-assisted birth.

21. A nurse is developing a plan of care for a newborn with omphalocele. Which of the following would the nurse include? A) Placing the newborn into a sterile drawstring bowel bag B) Using clean technique for dressing changes C) Preparing the newborn for incision and drainage D) Instituting gavage feedings

Ans: A Feedback: An infant with an omphalocele is placed in a sterile drawstring bowel bag that maintains a sterile environment for the exposed contents, allows visualization, reduces heat and moisture loss, and allows heat from radiant warmers to reach the newborn. The newborn is placed feet-first into the bag and the drawstring is secured around the torso. Strict sterile technique is necessary to prevent contamination of the exposed abdominal contents. An orogastric tube attached to low suction is used to prevent intestinal distention. IV therapy is administered to maintain fluid and electrolyte balance and provide a route for antibiotic therapy. Surgery is done to repair the defect, not incise and drain it.

7. An LGA newborn has a blood glucose level of 30 mg/dL and is exhibiting symptoms of hypoglycemia. Which of the following would the nurse do next? A) Administer intravenous glucose immediately. B) Feed the newborn 2 ounces of formula. C) Initiate blow-by oxygen therapy. D) Place the newborn under a radiant warmer.

Ans: A Feedback: If an LGA newborn's blood glucose level is below 40 mg/dL and is symptomatic, continuous infusion of parenteral glucose is needed. Supervised breast-feeding or formula feeding may be initial treatment options in asymptomatic hypoglycemia. Blow-by oxygen would have no effect on glucose levels; it may be helpful in promoting oxygenation. Placing the newborn under a radiant warmer would be a more appropriate measure for cold stress.

1. A newborn with severe meconium aspiration syndrome (MAS) is not responding to conventional treatment. Which of the following would the nurse anticipate as possibly necessary for this newborn? A) Extracorporeal membrane oxygenation (ECMO) B) Respiratory support with a ventilator C) Insertion of a laryngoscope for deep suctioning D) Replacement of an endotracheal tube via x-ray

Ans: A Feedback: If conventional measures are ineffective, then the nurse would need to prepare the newborn for ECMO. Hyperoxygenation, ventilatory support, and suctioning are typically used initially to promote tissue perfusion. However, if these are ineffective, ECMO would be the next step.

19. After determining that a newborn is in need of resuscitation, which of the following would the nurse do first? A) Dry the newborn thoroughly B) Suction the airway C) Administer ventilations D) Give volume expanders

Ans: A Feedback: If resuscitation is need, the nurse must first stabilize the newborn by drying the newborn thoroughly with a warm towel and provide warmth by placing him or her under a radiant heater to prevent rapid heat loss. Next the newborn's head is placed in a neutral position to open the airway and the airway is cleared with a bulb syringe or suction catheter. Breathing is stimulated. Often handling and rubbing the newborn with a dry towel may be all that is needed to stimulate respirations. Next ventilations and then chest compressions are done. Administration of epinephrine and/or volume expanders is the last step.

14. A woman who is 42 weeks pregnant comes to the clinic. Which of the following would be most important? A) Determining an accurate gestational age B) Asking her about the occurrence of contractions C) Checking for spontaneous rupture of membranes D) Measuring the height of the fundus

Ans: A Feedback: Incorrect dates account for the majority of prolonged or postterm pregnancies; many women have irregular menses and thus cannot identify the date of their last menstrual period accurately. Therefore, accurate gestational dating via ultrasound is essential. Asking about contractions and checking for ruptured membranes, although important assessments, would be done once the gestational age is confirmed. Measuring the height of the fundus would be unreliable because after 36 weeks, the fundal height drops due to lightening and may no longer correlate with gestational weeks.

9. While caring for a preterm newborn receiving oxygen therapy, the nurse monitors the oxygen therapy duration closely based on the understanding that the newborn is at risk for which of the following? A) Retinopathy of prematurity B) Metabolic acidosis C) Infection D) Cold stress

Ans: A Feedback: Oxygen therapy has been linked the pathogenesis of retinopathy of prematurity and is associated with the duration of oxygen use rather than the concentration of oxygen. Therefore, the nurse monitors the newborn's oxygen therapy closely. Metabolic acidosis may occur due to anaerobic metabolism used for heat production. Infection may occur for numerous reasons, but they are unrelated to oxygen therapy. Cold stress results from problems due to the preterm newborn's inadequate supply of brown fat, decreased muscle tone, and large body surface area.

17. After teaching the parents of a newborn with retinopathy of prematurity (ROP) about the disorder and treatment, which statement by the parents indicates that the teaching was successful? A) "Can we schedule follow-up eye examinations with the pediatric ophthalmologist now?" B) "We can fix the problem with surgery." C) "We'll make sure to administer eye drops each day for the next few weeks." D) "I'm sure the baby will grow out of it."

Ans: A Feedback: Parents of a newborn with suspected retinopathy of prematurity (ROP) should schedule follow-up vision screenings with a pediatric ophthalmologist every 2 to 3 weeks, depending on the severity of the findings at the initial examination.

16. The nurse is providing care to several pregnant women who may be scheduled for labor induction. The nurse identifies the woman with which Bishop score as having the best chance for a successful induction and vaginal birth? A) 11 B) 8 C) 6 D) 3

Ans: A Feedback: The Bishop score helps identify women who would be most likely to achieve a successful induction. The duration of labor is inversely correlated with the Bishop score: a score over 8 indicates a successful vaginal birth. Therefore the woman with a Bishop score of 11 would have the greatest chance for success. Bishop scores of less than 6 usually indicate that a cervical ripening method should be used prior to induction.

12. While reviewing a newborn's medical record, the nurse notes that the chest x-ray shows a ground glass pattern. The nurse interprets this as indicative of: A) Respiratory distress syndrome B) Transient tachypnea of the newborn C) Asphyxia D) Persistent pulmonary hypertension

Ans: A Feedback: The chest x-ray of a newborn with RDS reveals a reticular (ground glass) pattern. For TTN, the chest x-ray shows lung overaeration and prominent perihilar interstitial markings and streakings. A chest x-ray for asphyxia would reveal possible structural abnormalities that might interfere with respiration, but the results are highly variable. An echocardiogram would be done to evaluate persistent pulmonary hypertension.

29. A nurse is assisting in the resuscitation of a newborn. The nurse would expect to stop resuscitation efforts when the newborn has no heartbeat and respiratory effort after which time frame? A) 5 minutes B) 10 minutes C) 15 minutes D) 20 minutes

Ans: B Feedback: According to the American Heart Association and American Academy of Pediatrics Guidelines for Neonatal Resuscitation, resuscitation efforts may be stopped if the newborn exhibits no heartbeat and no respiratory effort after 10 minutes of continuous and adequate resuscitation.

26. While changing a female newborn's diaper, the nurse observes a mucus-like, slightly bloody vaginal discharge. Which of the following would the nurse do next? A) Document this as pseudomenstruation B) Notify the practitioner immediately C) Obtain a culture of the discharge D) Inspect for engorgement

Ans: A Feedback: The nurse should assess pseudomenstruation, a vaginal discharge composed of mucus mixed with blood, which may be present during the first few weeks of life. This discharge requires no treatment. The discharge is a normal finding and thus does not need to be reported immediately. It is not an indication of infection. The female genitalia normally will be engorged, so assessing for engorgement is not indicated.

21. The nurse is inspecting the external genitalia of a male newborn. Which of the following would alert the nurse to a possible problem? A) Limited rugae B) Large scrotum C) Palpable testes in scrotal sac D) Absence of engorgement

Ans: A Feedback: The scrotum usually appears relatively large and should be pink in white neonates and dark brown in neonates of color. Rugae should be well formed and should cover the scrotal sac. There should not be bulging, edema, or discoloration. Testes should be palpable in the scrotal sac and feel firm and smooth and be of equal size on both sides of the scrotal sac.

5. Which action would be most appropriate for the nurse to take when a newborn has an unexpected anomaly at birth? A) Show the newborn to the parents as soon as possible while explaining the defect. B) Remove the newborn from the birthing area immediately. C) Inform the parents that there is nothing wrong at the moment. D) Tell the parents that the newborn must go to the nursery immediately.

Ans: A Feedback: When an anomaly is identified at or after birth, parents need to be informed promptly and given a realistic appraisal of the severity of the condition, the prognosis, and treatment options so that they can participate in all decisions concerning their child. Removing the newborn from the area or telling them that the newborn needs to go to the nursery immediately is inappropriate and would only add to the parents' anxieties and fears. Telling them that nothing is wrong is inappropriate because it violates their right to know.

10. When planning the care for an SGA newborn, which action would the nurse determine as a priority? A) Preventing hypoglycemia with early feedings B) Observing for respiratory distress syndrome C) Promoting bonding between the parents and the newborn D) Monitoring vital signs every 2 hours

Ans: A Feedback: With the loss of the placenta at birth, the newborn must now assume control of glucose homeostasis. This is achieved by early oral intermittent feedings. Observing for respiratory distress, promoting bonding, and monitoring vital signs, although important, are not the priority for this newborn.

25. A group of nursing students are reviewing information about cesarean birth. The students demonstrate understanding of the information when they identify which of the following as an appropriate indication? (Select all that apply.) A) Active genital herpes infection B) Placenta previa C) Previous cesarean birth D) Prolonged labor E) Fetal distress

Ans: A, B, C, E Feedback: The leading indications for cesarean birth are previous cesarean birth, breech presentation, dystocia, and fetal distress. Examples of specific indications include active genital herpes, fetal macrosomia, fetopelvic disproportion, prolapsed umbilical cord, placental abnormality (placenta previa or abruptio placentae), previous classic uterine incision or scar, gestational hypertension, diabetes, positive HIV status, and dystocia. Fetal indications include malpresentation (nonvertex presentation), congenital anomalies (fetal neural tube defects, hydrocephalus, abdominal wall defects), and fetal distress.

19. After teaching a group of nursing students about tocolytic therapy, the instructor determines that the teaching was successful when they identify which drug as being used for tocolysis? (Select all that apply.) A) Nifedipine B) Terbutaline C) Dinoprostone D) Misoprostol E) Indomethacin

Ans: A, B, E Feedback: Medications most commonly used for tocolysis include magnesium sulfate (which reduces the muscle's ability to contract), terbutaline (Brethine, a beta-adrenergic), indomethacin (Indocin, a prostaglandin synthetase inhibitor), and nifedipine (Procardia, a calcium channel blocker). These drugs are used "off label": this means they are effective for this purpose but have not been officially tested and developed for this purpose by the FDA. Dinoprostone and misoprostol are used to ripen the cervix.

21. A nurse suspects that a preterm newborn is having problems with thermal regulation. Which of the following would support the nurse's suspicion? (Select all that apply.) A) Shallow, slow respirations B) Cyanotic hands and feet C) Irritability D) Hypertonicity E) Feeble cry

Ans: A, B, E Feedback: Typically, a preterm newborn that is having problems with thermal regulation is cool to cold to the touch. The hands, feet, and tongue may appear cyanotic. Respirations are shallow or slow, or signs of respiratory distress are present. The newborn is lethargic and hypotonic, feeds poorly, and has a feeble cry. Blood glucose levels are probably low, leading to hypoglycemia, due to the energy expended to keep warm

18. A nursing instructor is describing common problems associated with preterm birth. When describing the preterm newborn's risk for perinatal asphyxia, the instructor includes which of the following as contributing to the newborn's risk? (Select all that apply.) A) Surfactant deficiency B) Placental deprivation C) Immaturity of the respiratory control centers D) Decreased amounts of brown fat E) Depleted glycogen stores

Ans: A, C Feedback: Preterm newborns are at risk for perinatal asphyxia due to surfactant deficiency, unstable chest wall, immaturity of the respiratory control centers, small respiratory passages, and inability to clear mucus from the airways. Placental deprivation places the postterm newborn at risk for perinatal asphyxia. Decreased amounts of brown fat and depleted glycogen stores place the SGA newborn at risk for problems with thermoregulation.

22. A nurse is explaining to the parents of a child with bladder exstrophy about the care their infant requires. Which of the following would the nurse include in the explanation? (Select all that apply.) A) Covering the area with a sterile, clear, nonadherent dressing B) Irrigating the surface with sterile saline twice a day C) Monitoring drainage through the suprapubic catheter D) Administering prescribed antibiotic therapy E) Preparing for surgical intervention in about 2 weeks

Ans: A, C, D Feedback: Care for an infant with bladder exstrophy includes covering the area with a sterile, clear, nonadherent dressing and irrigating the bladder surface with sterile saline after each diaper change to prevent infection, assisting with insertion and monitoring drainage from suprapubic catheter, administering prescribed antibiotic therapy, and preparing the parents and infant for surgery within 48 hours after birth.

24. The nurse notifies the obstetrical team immediately because the nurse suspects that the pregnant woman may be exhibiting signs and symptoms of amniotic fluid embolism. Which findings would the nurse most likely assess? (Select all that apply.) A) Significant difficulty breathing B) Hypertension C) Tachycardia D) Pulmonary edema E) Bleeding with bruising

Ans: A, C, D, E Feedback: The clinical appearance is varied, but most women report difficulty breathing. Other symptoms include hypotension, cyanosis, seizures, tachycardia, coagulation failure, disseminated intravascular coagulation, pulmonary edema, uterine atony with subsequent hemorrhage, adult respiratory distress syndrome, and cardiac arrest

20. The nurse is assessing a newborn and suspects that the newborn was exposed to drugs in utero because the newborn is exhibiting signs of neonatal abstinence syndrome. Which of the following would the nurse expect to assess? (Select all that apply.) A) Tremors B) Diminished sucking C) Regurgitation D) Shrill, high-pitched cry E) Hypothermia F) Frequent sneezing

Ans: A, C, D, F Feedback: Signs and symptoms of neonatal abstinence syndrome include tremors, frantic sucking, regurgitation or projectile vomiting, shrill high-pitched cry, fever, and frequent sneezing.

22. The nurse is assessing a preterm newborn's fluid and hydration status. Which of the following would alert the nurse to possible overhydration? A) Decreased urine output B) Tachypnea C) Bulging fontanels D) Elevated temperature

Ans: C Feedback: Bulging fontanels in a preterm newborn suggest overhydration. Sunken fontanels, decreased urine output, and elevated temperature would suggest dehydration.

27. A newborn was diagnosed with a congenital heart defect and will undergo surgery at a later time. The nurse is teaching the parents about signs and symptoms that need to be reported. The nurse determines that the parents have understood the instructions when they state that they will report which of the following? (Select all that apply.) A) Weight loss B) Pale skin C) Fever D) Absence of edema E) Increased respiratory rate

Ans: A, C, E Feedback: Signs and symptoms that need to be reported include weight loss, poor feeding, cyanosis, breathing difficulties, irritability, increased respiratory rate, and fever.

25. A nurse is developing a plan of care for a preterm newborn to address the nursing diagnosis of risk for delayed development. Which of the following would the nurse include? (Select all that apply.) A) Clustering care to promote rest B) Positioning newborn in extension C) Using kangaroo care D) Loosely covering the newborn with blankets E) Providing nonnutritive sucking

Ans: A, C, E Feedback: The nurse would focus the plan of care on developmental care, which includes clustering care to promote rest and conserve energy, using flexed positioning to simulate in utero positioning, using kangaroo care to promote skin to skin sensations, swaddling with a blanket to maintain the flexed position, and providing nonnutritive sucking.

16. A nurse is assessing a newborn who has been classified as small for gestational age. Which of the following would the nurse expect to find? (Select all that apply.) A) Wasted extremity appearance B) Increased amount of breast tissue C) Sunken abdomen D) Adequate muscle tone over buttocks E) Narrow skull sutures

Ans: A, C, E Feedback: Typical characteristics of SGA newborns include a head that is disproportionately large compared to the rest of the body, wasted appearance of the extremities, reduced subcutaneous fat stores, decreased amount of breast tissue, scaphoid abdomen, wide skull sutures, poor muscle tone over buttocks and cheeks, loose and dry skin appearing oversized, and a thin umbilical cord.

11. A woman gives birth to a newborn at 36 weeks' gestation. She tells the nurse, "I'm so glad that my baby isn't premature." Which response by the nurse would be most appropriate? A) "You are lucky to have given birth to a term newborn." B) "We still need to monitor him closely for problems." C) "How do you feel about delivering your baby at 36 weeks?" D) "Your baby is premature and needs monitoring in the NICU."

Ans: B Feedback: A baby born at 36 weeks' gestation is considered a late preterm newborn. These newborns face similar challenges as those of preterm newborns and require similar care. Telling the mother that close monitoring is necessary can prevent any misconceptions that she might have and prepare her for what might arise. The baby is not considered a term newborn, nor is the baby considered premature. The decision for care in the NICU would depend on the newborn's status. Asking the woman how she feels about the delivery demonstrates caring but does not address the woman's lack of understanding about her newborn.

17. The nurse is reviewing the medical record of a newborn born 2 hours ago. The nurse notes that the newborn was delivered at 35 weeks' gestation. The nurse would classify this newborn as which of the following? A) Preterm B) Late preterm C) Full term D) Postterm

Ans: B Feedback: A late preterm infant is one born between 34 to 36 6/7 weeks of gestation. A preterm infant is one born before 37 completed weeks' gestation. A full-term infant is one born between 38 to 41 weeks' gestation. A postterm newborn is one born at 42 weeks' gestation or later.

15. When describing newborns with birth-weight variations to a group of nursing students, the instructor identifies which variation if the newborn weighs 5.2 lb at any gestational age? A) Small for gestational age B) Low birth weight C) Very low birth weight D) Extremely low birth weight

Ans: B Feedback: A low-birth-weight newborn weighs less than 5.5 lb (2,500 g) but more than 3 lb 5 oz. A very-low-birth-weight newborn would weigh less than 3 lb 5 oz but more than 2 lb 3 oz (1,000 g). An extremely-low-birth-weight newborn weighs less than 2 lb 3 oz (1,000g). A small-for-gestational-age newborn typically weighs less than 5 lb 8 oz (2,500 g) at term.

11. A newborn has an Apgar score of 6 at 5 minutes. Which of the following is the priority? A) Initiating IV fluid therapy B) Beginning resuscitative measures C) Promoting kangaroo care D) Obtaining a blood culture

Ans: B Feedback: An Apgar score below 7 at 1 or 5 minutes indicates the need for resuscitation. Intravenous fluid therapy and blood cultures may be done once resuscitation is started. Kangaroo care would be appropriate once the newborn is stable.

13. A woman with preterm labor is receiving magnesium sulfate. Which finding would require the nurse to intervene immediately? A) Respiratory rate of 16 breaths per minute B) Diminished deep tendon reflexes C) Urine output of 45 mL/hour D) Alert level of consciousness

Ans: B Feedback: Diminished deep tendon reflexes suggest magnesium toxicity, which requires immediate intervention. Additional signs of magnesium toxicity include a respiratory rate less than 12 breaths/minute, urine output less than 30 mL/hour, and a decreased level of consciousness.

3. Which of the following would the nurse include in the plan of care for a newborn receiving phototherapy? A) Keeping the newborn in the supine position B) Covering the newborn's eyes while under the bililights C) Ensuring that the newborn is covered or clothed D) Reducing the amount of fluid intake to 8 ounces daily

Ans: B Feedback: During phototherapy, the newborn's eyes are covered to protect them from the lights. The newborn is turned every 2 hours to expose all areas of the body to the lights and is kept undressed, except for the diaper area, to provide maximum body exposure to the lights. Fluid intake is increased to allow for added fluid, protein, and calories.

22. A nurse is describing the risks associated with prolonged pregnancies as part of an inservice presentation. Which of the following would the nurse be least likely to incorporate in the discussion as an underlying reason for problems in the fetus? A) Aging of the placenta B) Increased amniotic fluid volume C) Meconium aspiration D) Cord compression

Ans: B Feedback: Fetal risks associated with a prolonged pregnancy include macrosomia, shoulder dystocia, brachial plexus injuries, low Apgar scores, postmaturity syndrome (loss of subcutaneous fat and muscle and meconium staining), and cephalopelvic disproportion. All of these conditions predispose this fetus to birth trauma or a surgical birth. Uteroplacental insufficiency, meconium aspiration, and intrauterine infection contribute to the increased rate of perinatal deaths (Beacock, 2011). As the placenta ages, its perfusion decreases and it becomes less efficient at delivering oxygen and nutrients to the fetus. Amniotic fluid volume also begins to decline by 40 weeks of gestation, possibly leading to oligohydramnios, subsequently resulting in fetal hypoxia and an increased risk of cord compression because the cushioning effect offered by adequate fluid is no longer present. Hypoxia and oligohydramnios predispose the fetus to aspiration of meconium, which is released by the fetus in response to a hypoxic insult (Caughey & Butler, 2010).

6. The nurse prepares to administer a gavage feeding for a newborn with transient tachypnea based on the understanding that this type of feeding is necessary for which reason? A) Lactase enzymatic activity is not adequate. B) Oxygen demands need to be reduced. C) Renal solute lead must be considered. D) Hyperbilirubinemia is likely to develop.

Ans: B Feedback: For the newborn with transient tachypnea, the newborn's respiratory rate is high, increasing his oxygen demand. Thus, measures are initiated to reduce this demand. Gavage feedings are one way to do so. With transient tachypnea, enzyme activity and kidney function are not affected. This condition typically resolves within 72 hours. The risk for hyperbilirubinemia is not increased.

6. The fetus of a woman in labor is determined to be in persistent occiput posterior position. Which of the following would the nurse identify as the priority intervention? A) Position changes B) Pain relief measures C) Immediate cesarean birth D) Oxytocin administration

Ans: B Feedback: Intense back pain is associated with persistent occiput posterior position. Therefore, a priority is to provide pain relief measures. Position changes that can promote fetal head rotation are important after the nurse institutes pain relief measures. Additionally, the woman's ability to cooperate and participate in these position changes is enhanced when she is experiencing less pain. Immediate cesarean birth is not indicated unless there is evidence of fetal distress. Oxytocin would add to the woman's already high level of pain.

6. The nurse prepares to assess a newborn who is considered to be large for gestational age (LGA). Which of the following would the nurse correlate with this gestational age variation? A) Strong, brisk motor skills B) Difficulty in arousing to a quiet alert state C) Birth weight of 7 lb 14 oz D) Wasted appearance of extremities

Ans: B Feedback: LGA newborns typically are more difficult to arouse to a quiet alert state. They have poor motor skills, have a large body that appears plump and full-sized, and usually weigh more than 8 lb 13 oz at term.

23. A group of nursing students are reviewing information about methods used for cervical ripening. The students demonstrate understanding of the information when they identify which of the following as a mechanical method? A) Herbal agents B) Laminaria C) Membrane stripping D) Amniotomy

Ans: B Feedback: Laminaria is a hygroscopic dilator that is used as a mechanical method for cervical ripening. Herbal agents are a nonpharmacologic method. Membrane stripping and amniotomy are considered surgical methods.

15. Which of the following would not be considered a risk factor for bronchopulmonary dysplasia (chronic lung disease)? A) Preterm birth (less than 32 weeks) B) Female gender C) White race D) Sepsis

Ans: B Feedback: Male gender is more commonly associated with bronchopulmonary dysplasia. Preterm birth of less than 32 weeks' gestation, sepsis, white race, excessive fluid intake during the first few days of life, severe RDS with mechanical ventilation for more than 1 week, and patent ductus arteriosus are all risk factors associated with chronic lung disease in the newborn.

13. Which of the following, if noted in the maternal history, would the nurse identify as possibly contributing to the birth of an LGA newborn? A) Drug abuse B) Diabetes C) Preeclampsia D) Infection

Ans: B Feedback: Maternal factors that increase the chance of having an LGA newborn include maternal diabetes mellitus or glucose intolerance, multiparity, prior history of a macrosomic infant, postdated gestation, maternal obesity, male fetus, and genetics. Drug abuse is associated with SGA newborns and preterm newborns. A maternal history of preeclampsia and infection would be associated with preterm birth.

7. Which of the following would the nurse include when teaching a new mother about the difference between pathologic and physiologic jaundice? A) Physiologic jaundice results in kernicterus. B) Pathologic jaundice appears within 24 hours after birth. C) Both are treated with exchange transfusions of maternal O- blood. D) Physiologic jaundice requires transfer to the NICU.

Ans: B Feedback: Pathologic jaundice appears within 24 hours after birth, whereas physiologic jaundice commonly appears around the third to fourth days of life. Kernicterus is more commonly associated with pathologic jaundice. An exchange transfusion is used only if the total serum bilirubin level remains elevated after intensive phototherapy. With this procedure, the newborn's blood is removed and replaced with nonhemolyzed red blood cells from a donor. Physiologic jaundice often is treated at home.

10. After teaching the parents of a newborn with periventricular hemorrhage about the disorder and treatment, which statement by the parents indicates that the teaching was successful? A) "We'll make sure to cover both of his eyes to protect them." B) "Our newborn could develop a learning disability later on." C) "Once the bleeding ceases, there won't be any more worries." D) "We need to get family members to donate blood for transfusion."

Ans: B Feedback: Periventricular-intraventricular hemorrhage has long-term sequelae such as seizures, hydrocephalus, periventricular leukomalacia, cerebral palsy, learning disabilities, vision or hearing deficits, and mental retardation. Covering the eyes is more appropriate for the newborn receiving phototherapy. The bleeding in the brain can lead to serious long-term effects. Blood transfusions are not used to treat periventricular hemorrhage.

23. A nurse is teaching new parents about bathing their newborn. The nurse determines that the teaching was successful when the parents state which of the following? A) "We can put a tiny bit of lotion on his skin and then rub it in gently." B) "We should avoid using any kind of baby powder." C) "We need to bathe him at least four to five times a week." D) "We should clean his eyes after washing his face and hair."

Ans: B Feedback: Powders should not be used because they can be inhaled, causing respiratory distress. If the parents want to use oils and lotions, have them apply a small amount onto their hand first, away from the newborn; this warms the lotion. Then the parents should apply the lotion or oil sparingly. Parents need to be instructed that a bath two or three times weekly is sufficient for the first year because too frequent bathing may dry the skin. The eyes are cleaned first and only with plain water; then the rest of the face is cleaned with plain water.

3. The nurse would be alert for possible placental abruption during labor when assessment reveals which of the following? A) Macrosomia B) Gestational hypertension C) Gestational diabetes D) Low parity

Ans: B Feedback: Risk factors for placental abruption include preeclampsia, gestational hypertension, seizure activity, uterine rupture, trauma, smoking, cocaine use, coagulation defects, previous history of abruption, domestic violence, and placental pathology. Macrosomia, gestational diabetes, and low parity are not considered risk factors.

28. A newborn is scheduled to undergo a screening test for phenylketonuria (PKU). The nurse prepares to obtain the blood sample from the newborn's: A) Finger B) Heel C) Scalp vein D) Umbilical vein

Ans: B Feedback: Screening tests for genetic and inborn errors of metabolism require a few drops of blood taken from the newborn's heel. The finger, scalp vein, and umbilical vein are inappropriate sites for the blood sample.

8. When planning the care of a newborn addicted to cocaine who is experiencing withdrawal, which of the following would be least appropriate to include? A) Wrapping the newborn snugly in a blanket B) Waking the newborn every hour C) Checking the newborn's fontanels D) Offering a pacifier

Ans: B Feedback: Stimuli need to be decreased. Waking the newborn every hour would most likely be too stimulating. Measures such as swaddling the newborn tightly and offering a pacifier help to decrease irritable behaviors. A pacifier also helps to satisfy the newborn's need for nonnutritive sucking. Checking the fontanels provides evidence of hydration.

16. A group of nursing students are reviewing the different types of congenital heart disease in infants. The students demonstrate a need for additional review when they identify which of the following as an example of increased pulmonary blood flow (left-to-right shunting)? A) Atrial septal defect B) Tetralogy of Fallot C) Ventricular septal defect D) Patent ductus arteriosus

Ans: B Feedback: Tetralogy of Fallot is a congenital heart condition that results from decreased, not increased, pulmonary blood flow. Atrial septal defect, ventricular septal defect, and patent ductus arteriosus are heart conditions that involve increased blood flow from higher pressure (left side of heart) to lower pressure (right side of heart), resulting in left-to-right shunting.

25. A nurse is teaching a postpartum client and her partner about caring for their newborn's umbilical cord site. Which statement by the parents indicates a need for additional teaching? A) "We can put him in the tub to bathe him once the cord falls off and is healed." B) "The cord stump should change from brown to yellow." C) "Exposing the stump to the air helps it to dry." D) "We need to call the doctor if we notice a funny odor."

Ans: B Feedback: The cord stump should change color from yellow to brown or black. Therefore the parents need additional teaching if they state the color changes from brown to yellow. Tub baths are avoided until the cord has fallen off and the area is healed. Exposing the stump to the air helps it to dry. The parents should notify their primary care provider if there is any bleeding, redness, drainage, or foul odor from the cord stump.

19. Assessment of a newborn's head circumference reveals that it is 34 cm. The nurse would suspect that this newborn's chest circumference would be: A) 30 cm B) 32 cm C) 34 cm D) 36 cm

Ans: B Feedback: The newborn's chest should be round, symmetric, and 2 to 3 cm smaller than the head circumference. Therefore, this newborn's chest circumference would be 31 to 32 cm to be normal.

20. The nurse is auscultating a newborn's heart and places the stethoscope at the point of maximal impulse at which location? A) Just superior to the nipple, at the midsternum B) Lateral to the midclavicular line at the fourth intercostal space C) At the fifth intercostal space to the left of the sternum D) Directly adjacent to the sternum at the second intercostals space

Ans: B Feedback: The point of maximal impulse (PMI) in a newborn is a lateral to midclavicular line located at the fourth intercostal space.

27. A nursing instructor is describing the advantages and disadvantages associated with newborn circumcision to a group of nursing students. Which statement by the students indicates effective teaching? A) "Sexually transmitted infections are more common in circumcised males." B) "The rate of penile cancer is less for circumcised males." C) "Urinary tract infections are more easily treated in circumcised males." D) "Circumcision is a risk factor for acquiring HIV infection."

Ans: B Feedback: The risk for penile cancer appears to be slightly lower for males who are circumcised. However, penile cancer is rare and other risk factors such as genital warts and HPV infection seem to play a larger role. Sexually transmitted infections are less common in circumcised males but the risk is believed to be related more to behavioral factors than circumcision status. Circumcised males have a 50% lower risk of acquiring HIV infection. Urinary tract infections are slightly less common in circumcised boys. However, rates are low in both circumcised and uncircumcised boys and are easily treated without long-term sequelae.

2. A primigravida whose labor was initially progressing normally is now experiencing a decrease in the frequency and intensity of her contractions. The nurse would assess the woman for which condition? A) A low-lying placenta B) Fetopelvic disproportion C) Contraction ring D) Uterine bleeding

Ans: B Feedback: The woman is experiencing dystocia most likely due to hypotonic uterine dysfunction and fetopelvic disproportion associated with a large fetus. A low-lying placenta, contraction ring, or uterine bleeding would not be associated with a change in labor pattern.

28. When developing the plan of care for a newborn with an acquired condition, which of the following would the nurse include to promote participation by the parents? A) Use verbal instructions primarily for explanations B) Assist with decision making process C) Provide personal views about their decisions D) Encourage them to refrain from showing emotions

Ans: B Feedback: To promote parental participation, the nurse should assist them with making decisions about treatment, and support their decisions for the newborn's care. Imposing personal views about their decisions is inappropriate and undermines the nurse-client relationship. In addition, the nurse would assess their ability to cope with the diagnosis, encourage them to verbalize their feelings about the newborn's condition and treatment and educate them about the newborn's condition using written information and pictures to enhance understanding.

29. Assessment of a newborn reveals transient tachypnea. The nurse reviews the newborn's medical record. Which of the following would the nurse be least likely to identify as a risk factor for this condition? A) Cesarean birth B) Shortened labor C) Central nervous system depressant during labor D) Maternal asthma

Ans: B Feedback: Transient tachypnea of the newborn occurs when the fetal liquid in the lungs is removed slowly or incompletely. This can be due to the lack of thoracic squeezing that occurs during a cesarean birth or diminished respiratory effort if the mother received central nervous system depressant medication. Prolonged labor, macrosomia of the fetus and maternal asthma also have been associated with this condition.

15. After teaching a couple about what to expect with their planned cesarean birth, which statement indicates the need for additional teaching? A) "Holding a pillow against my incision will help me when I cough." B) "I'm going to have to wait a few days before I can start breast-feeding." C) "I guess the nurses will be getting me up and out of bed rather quickly." D) "I'll probably have a tube in my bladder for about 24 hours or so."

Ans: B Feedback: Typically, breast-feeding is initiated early as soon as possible after birth to promote bonding. The woman may need to use alternate positioning techniques to reduce incisional discomfort. Splinting with pillows helps to reduce the discomfort associated with coughing. Early ambulation is encouraged to prevent respiratory and cardiovascular problems and promote peristalsis. An indwelling urinary catheter is typically inserted to drain the bladder. It usually remains in place for approximately 24 hours.

10. When assessing several women for possible VBAC, which woman would the nurse identify as being the best candidate? A) One who has undergone a previous myomectomy B) One who had a previous cesarean birth via a low transverse incision C) One who has a history of a contracted pelvis D) One who has a vertical incision from a previous cesarean birth

Ans: B Feedback: VBAC is an appropriate choice for women who have had a previous cesarean birth with a lower abdominal transverse incision. It is contraindicated in women who have a prior classic uterine incision (vertical), prior transfundal surgery, such as myomectomy, or a contracted pelvis.

12. Which of the following would be most appropriate for the nurse to do when assisting parents who have experienced the loss of their preterm newborn? A) Avoid using the terms "death" or "dying." B) Provide opportunities for them to hold the newborn. C) Refrain from initiating conversations with the parents. D) Quickly refocus the parents to a more pleasant topic.

Ans: B Feedback: When dealing with grieving parents, nurses should provide them with opportunities to hold the newborn if they desire. In addition, the nurse should provide the parents with as many memories as possible, encouraging them to see, touch, dress, and take pictures of the newborn. These interventions help to validate the parents' sense of loss, relive the experience, and attach significance to the meaning of loss. The nurse should use appropriate terminology, such as "dying," "died," and "death," to help the parents accept the reality of the death. Nurses need to demonstrate empathy and to respect the parents' feelings, responding to them in helpful and supportive ways. Active listening and allowing the parents to vent their frustrations and anger help validate the parents' feelings and facilitate the grieving process.

18. The nurse is assessing a newborn's eyes. Which of the following would the nurse identify as normal? (Select all that apply.) A) Slow blink response B) Able to track object to midline C) Transient deviation of the eyes D) Involuntary repetitive eye movement E) Absent red reflex

Ans: B, C, D Feedback: Assessment of the eyes should reveal a rapid blink reflex, ability to track objects to the midline, transient strabismus (deviation or wandering of the eyes independently), searching nystagmus (involuntary repetitive eye movement), and a red reflex.

18. The nurse is assessing the newborn of a mother who had gestational diabetes. Which of the following would the nurse expect to find? (Select all that apply.) A) Pale skin color B) Buffalo hump C) Distended upper abdomen D) Excessive subcutaneous fat E) Long slender neck

Ans: B, C, D Feedback: Infants of diabetic mothers exhibit full rosy cheeks with a ruddy skin color, short neck, buffalo hump over the nape of the neck, massive shoulders, distended upper abdomen, and excessive subcutaneous fat tissue.

30. A newborn is diagnosed with meconium aspiration syndrome. When assessing this newborn, which of the following would the nurse expect to find? (Select all that apply.) A) Pigeon chest B) Prolonged tachypnea C) Intercostal retractions D) High blood pH level E) Coarse crackles on auscultation

Ans: B, C, E Feedback: Assessment findings associated with meconium aspiration syndrome include barrel-shaped chest with an increased anterior-posterior (AP) chest diameter (similar to that found in a client with chronic obstructive pulmonary disease), prolonged tachypnea, progression from mild to severe respiratory distress, intercostal retractions, end-expiratory grunting, and cyanosis. Coarse crackles and rhonchi are noted on lung auscultation.

9. A newborn is suspected of having fetal alcohol syndrome. Which of the following would the nurse expect to assess? A) Bradypnea B) Hydrocephaly C) Flattened maxilla D) Hypoactivity

Ans: C Feedback: A newborn with fetal alcohol syndrome exhibits characteristic facial features such as microcephaly (not hydrocephaly), small palpebral fissures, and abnormally small eyes, flattened or absent maxilla, epicanthal folds, thin upper lip, and missing vertical groove in the median portion of the upper lip. Bradypnea is not typically associated with fetal alcohol syndrome. Fine and gross motor development is delayed, and the newborn shows poor hand-eye coordination but not hypoactivity.

2. When assessing a postterm newborn, which of the following would the nurse correlate with this gestational age variation? A) Moist, supple, plum skin appearance B) Abundant lanugo and vernix C) Thin umbilical cord D) Absence of sole creases

Ans: C Feedback: A postterm newborn typically exhibits a thin umbilical cord; dry, cracked, wrinkled skin; limited vernix and lanugo; and creases covering the entire soles of the feet.

5. The nurse frequently assesses the respiratory status of a preterm newborn based on the understanding that the newborn is at increased risk for respiratory distress syndrome because of which of the following? A) Inability to clear fluids B) Immature respiratory control center C) Deficiency of surfactant D) Smaller respiratory passages

Ans: C Feedback: A preterm newborn is at increased risk for respiratory distress syndrome (RDS) most commonly because of a surfactant deficiency. Surfactant helps to keep the alveoli open and maintain lung expansion. With a deficiency, the alveoli collapse, predisposing the newborn to RDS. An inability to clear fluids can lead to transient tachypnea. Immature respiratory control centers lead to an increased risk for apnea. Smaller respiratory passages lead to an increased risk for obstruction.

13. A newborn is suspected of developing persistent pulmonary hypertension. The nurse would expect to prepare the newborn for which of the following to confirm the suspicion? A) Chest x-ray B) Blood cultures C) Echocardiogram D) Stool for occult blood

Ans: C Feedback: An echocardiogram is used to reveal right-to-left shunting of blood to confirm the diagnosis of persistent pulmonary hypertension. Chest x-ray would be most likely used to aid in the diagnosis of RDS or TTN. Blood cultures would be helpful in evaluating for neonatal sepsis. Stool for occult blood may be done to evaluate for NEC.

19. The nurse is assessing a newborn who is large for gestational age. The newborn was born breech. The nurse suspects that the newborn may have experienced trauma to the upper brachial plexus based on which assessment findings? A) Absent grasp reflex B) Hand weakness C) Absent Moro reflex D) Facial asymmetry

Ans: C Feedback: An injury to the upper brachial plexus, or Erb's palsy, is manifested by adduction, pronation, and internal rotation of the affected extremity, absent shoulder movement, absent Moro reflex and positive grasp reflex. An absent grasp reflex and hand weakness is noted with a lower brachial plexus injury. Facial asymmetry is associated with a cranial nerve injury.

8. A group of pregnant women are discussing high-risk newborn conditions as part of a prenatal class. When describing the complications that can occur in these newborns to the group, which would the nurse include as being at lowest risk? A) Small-for-gestational-age (SGA) newborns B) Large-for-gestational-age (LGA) newborns C) Appropriate-for-gestational-age (AGA) newborns D) Low-birth-weight newborns

Ans: C Feedback: Appropriate-for-gestational-age (AGA) newborns are at the lowest risk for any problems. AGA characterizes approximately 80% of newborns and describes a newborn with a normal length, weight, head circumference, and body mass index. The other categories all have an increased risk of complications.

24. A group of nursing students are reviewing the literature in preparation for a class presentation on newborn pain prevention and management. Which of the following would the students be most likely to find about this topic? A) Newborn pain is frequently recognized and treated B) Newborns rarely experience pain with procedures C) Pain is frequently mistaken for irritability or agitation D) Newborns may be less sensitive to pain than adult.

Ans: C Feedback: Assessment of pain in the newborn remains a contentious and vexing problem. According to an international consortium, principles of newborn pain prevention and management include the following: newborn pain frequently goes unrecognized and undertreated; newborns experience pain and analgesics should be given; a procedure considered painful for an adults should also be considered painful for a newborn; newborns may be more sensitive to pain than adults; and pain behavior is frequently mistaken for irritability and agitation.

23. A nursing student is preparing a presentation for the class on clubfoot. The student determines that the presentation was successful when the class states which of the following? A) Clubfoot is a common genetic disorder. B) The condition affects girls more often than boys. C) The exact cause of clubfoot is not known. D) The intrinsic form can be manually reduced.

Ans: C Feedback: Clubfoot is a complex, multifactorial deformity with genetic and intrauterine factors. Heredity and race seem to factor into the incidence, but the means of transmission and the etiology are unknown. Most newborns with clubfoot have no identifiable genetic, syndromal, or extrinsic cause. Clubfoot affects boys twice as often as girls. With the intrinsic type, manual reduction is not possible.

18. A nurse is preparing an inservice education program for a group of nurses about dystocia involving problems with the passenger. Which of the following would the nurse most likely include as the most common problem? A) Macrosomia B) Breech presentation C) Persistent occiput posterior position D) Multifetal pregnancy

Ans: C Feedback: Common problems involving the passenger include occiput posterior position, breech presentation, multifetal pregnancy, excessive size (macrosomia) as it relates to cephalopelvic disproportion (CPD), and structural anomalies. Of these, persistent occiput posterior is the most common malposition, occurring in about 15% of laboring women.

5. A woman in labor is experiencing hypotonic uterine dysfunction. Assessment reveals no fetopelvic disproportion. Which group of medications would the nurse expect to administer? A) Sedatives B) Tocolytics C) Oxytocin D) Corticosteroids

Ans: C Feedback: For hypotonic labor, a uterine stimulant such as oxytocin may be ordered once fetopelvic disproportion is ruled out. Sedatives might be helpful for the woman with hypertonic uterine contractions to promote rest and relaxation. Tocolytics would be ordered to control preterm labor. Corticosteroids may be given to enhance fetal lung maturity for women experiencing preterm labor.

20. A nurse is developing a plan of care for a preterm infant experiencing respiratory distress. Which of the following would the nurse be least likely to include in this plan? A) Stimulate the infant with frequent handling. B) Keep the newborn in a warmed isolette. C) Administer oxygen using an oxygen hood. D) Give gavage or continuous tube feedings.

Ans: C Feedback: For the preterm infant experiencing respiratory distress, the nurse would expect to handle the newborn as little as possible to reduce oxygen requirements. Other appropriate interventions include keeping the infant warm, preferably in a warmed isolette to conserve the baby's energy and prevent cold stress; administer oxygen using an oxygen hood; and provide energy through calories via intravenous dextrose or gavage or continuous tube feedings to prevent hypoglycemia.

26. A nurse is teaching the mother of a newborn diagnosed with galactosemia about dietary restrictions. The nurse determines that the mother has understood the teaching when she identifies which of the following as needing to be restricted? A) Phenylalanine B) Protein C) Lactose D) Iodine

Ans: C Feedback: Lifelong restriction of lactose is required for galactosemia. Phenylalanine is restricted for those with phenylketonuria. Low protein is needed with maple syrup urine disease. Iodine would not be restricted for any inborn error of metabolism

2. Which of the following would the nurse expect to assess in a newborn who develops sepsis? A) Increased urinary output B) Interest in feeding C) Hypothermia D) Wakefulness

Ans: C Feedback: Manifestations of sepsis are typically nonspecific and may include hypothermia (temperature instability), oliguria or anuria, lack of interest in feeding, and lethargy.

25. A group of students are reviewing information about the effects of substances on the newborn. The students demonstrate understanding of the information when they identify which drug as not being associated with teratogenic effects on the fetus? A) Alcohol B) Nicotine C) Marijuana D) Cocaine

Ans: C Feedback: Marijuana has not been shown to have teratogenic effects on the fetus. Alcohol, nicotine and cocaine do affect the fetus.

24. A new mother who is breast-feeding her newborn asks the nurse, "How will I know if my baby is drinking enough?" Which response by the nurse would be most appropriate? A) "If he seems content after feeding, that should be a sign." B) "Make sure he drinks at least 5 minutes on each breast." C) "He should wet between 6 to 12 diapers each day." D) "If his lips are moist, then he's okay."

Ans: C Feedback: Soaking 6 to 12 diapers a day indicates adequate hydration. Contentedness after feeding is not an indicator for adequate hydration. Typically a newborn wakes up 8 to 12 times per day for feeding. As the infant gets older, the time on the breast increases. Moist mucous membranes help to suggest adequate hydration but this is not the best indicator.

20. A nurse is assessing a pregnant woman who has come to the clinic. The woman reports that she feels some heaviness in her thighs since yesterday. The nurse suspects that the woman may be experiencing preterm labor based on which additional assessment finding? A) Dull low backache B) Malodorous vaginal discharge C) Dysuria D) Constipation

Ans: C Feedback: Symptoms of preterm labor are often subtle and may include change or increase in vaginal discharge with mucus, water, or blood in it; pelvic pressure; low, dull backache; nausea, vomiting or diarrhea, and intestinal cramping with or without diarrhea.

3. The parents of a preterm newborn being cared for in the neonatal intensive care unit (NICU) are coming to visit for the first time. The newborn is receiving mechanical ventilation and intravenous fluids and medications and is being monitored electronically by various devices. Which action by the nurse would be most appropriate? A) Suggest that the parents stay for just a few minutes to reduce their anxiety. B) Reassure them that their newborn is progressing well. C) Encourage the parents to touch their preterm newborn. D) Discuss the care they will be giving the newborn upon discharge.

Ans: C Feedback: The NICU environment can be overwhelming. Therefore, the nurse should address their reactions and explain all the equipment being used. On entering the NICU, the nurse should encourage the parents to touch, interact, and hold their newborn. Doing so helps to acquaint the parents with their newborn, promotes self-confidence, and fosters parent-newborn attachment. The parents should be allowed to stay for as long as they feel comfortable. Reassurance, although helpful, may be false reassurance at this time. Discussing discharge care can be done later once the newborn's status improves and plans for discharge are initiated.

24. Assessment of newborn reveals a large protruding tongue, slow reflexes, distended abdomen, poor feeding, hoarse cry, goiter and dry skin. Which of the following would the nurse suspect? A) Phenylketonuria B) Galactosemia C) Congenital hypothyroidism D) Maple syrup urine disease

Ans: C Feedback: The manifestations listed correlate with congenital hypothyroidism. With phenylketonuria, the infant appears normal at birth but by 6 months of age, signs of slow mental development are evident. Vomiting, poor feeding, failure to thrive, overactivity and musty-smelling urine are additional signs. With maple syrup urine disease, signs and symptoms include lethargy, poor feeding, vomiting, weight loss, seizures, shrill cry, shallow respirations, loss of reflexes, and a sweet maple syrup odor to the urine. With galactosemia, manifestations include vomiting, hypoglycemia, hyperbilirubinemia, poor weight gain, cataracts, and frequent infections.

7. A woman gave birth to a newborn via vaginal delivery with the use of a vacuum extractor. The nurse would be alert for which of the following in the newborn? A) Asphyxia B) Clavicular fracture C) Caput succedaneum D) Central nervous system injury

Ans: C Feedback: Use of forceps or a vacuum extractor poses the risk of tissue trauma, such as ecchymoses, facial and scalp lacerations, facial nerve injury, cephalhematoma, and caput succedaneum. Asphyxia may be related to numerous causes but it is not associated with use of a vacuum extractor. Clavicular fracture is associated with shoulder dystocia. Central nervous system injury is not associated with the use of a vacuum extractor.

9. A woman with a history of crack cocaine abuse is admitted to the labor and birth area. While caring for the client, the nurse notes a sudden onset of fetal bradycardia. Inspection of the abdomen reveals an irregular wall contour. The client also complains of acute abdominal pain that is continuous. Which of the following would the nurse suspect? A) Amniotic fluid embolism B) Shoulder dystocia C) Uterine rupture D) Umbilical cord prolapse

Ans: C Feedback: Uterine rupture is associated with crack cocaine use, and generally the first and most reliable sign is sudden fetal distress accompanied by acute abdominal pain, vaginal bleeding, hematuria, irregular wall contour, and loss of station in the fetal presenting part. Amniotic fluid embolism often is manifested with a sudden onset of respiratory distress. Shoulder dystocia is noted when continued fetal descent is obstructed after the fetal head is delivered. Umbilical cord prolapse is noted as the protrusion of the cord alongside or ahead of the presenting part of the fetus.

17. After teaching a group of nursing students about risk factors associated with dystocia, the instructor determines that the teaching was successful when the students identify which of the following as increasing the risk? (Select all that apply.) A) Pudendal block anesthetic use B) Multiparity C) Short maternal stature D) Maternal age over 35 E) Breech fetal presentation

Ans: C, D, E Feedback: According to the American College of Obstetrics and Gynecology (ACOG, 2009a), factors associated with an increased risk for dystocia include epidural analgesia, excessive analgesia, multiple gestation, hydramnios, maternal exhaustion, ineffective maternal pushing technique, occiput posterior position, longer first stage of labor, nulliparity, short maternal stature (less than 5 feet tall), fetal birth weight (more than 8.8 lb), shoulder dystocia, abnormal fetal presentation or position (breech), fetal anomalies (hydrocephalus), maternal age older than 35 years, high caffeine intake, overweight, gestational age more than 41 weeks, chorioamnionitis, ineffective uterine contractions, and high fetal station at complete cervical dilation.

30. A nurse is providing teaching to a new mother about her newborn's nutritional needs. Which of the following would the nurse be most likely to include in the teaching? (Select all that apply.) A) Supplementing with iron if the woman is breast-feeding B) Providing supplemental water intake with feedings C) Feeding the newborn every 2 to 4 hours during the day D) Burping the newborns frequently throughout each feeding E) Using feeding time for promoting closeness

Ans: C, D, E Feedback: Most newborns are on demand feeding schedules and are allowed to feed when they awaken. When they go home, mothers are encouraged to feed their newborns every 2 to 4 hours during the day and only when the newborn awakens during the night for the first few days after birth. Newborns swallow air during feedings, which causes discomfort and fussiness. Parents can prevent this by burping them frequently throughout the feeding. Feeding is also more than an opportunity to get nutrients into the newborn. It is also a time for closeness and sharing. Iron supplementation is recommended for infants who are bottle-fed. Fluid requirements for the newborn and infant do range from 100 to 150 mL/kg daily. This requirement can be met through breast or bottle feedings. Thus, additional water supplementation is not necessary.

4. Assessment of a woman in labor who is experiencing hypertonic uterine dysfunction would reveal contractions that are: A) Well coordinated B) Poor in quality C) Rapidly occurring D) Erratic

Ans: D Feedback: Hypertonic contractions occur when the uterus never fully relaxes between contractions, making the contractions erratic and poorly coordinated because more than one uterine pacemaker is sending signals for contraction. Hypotonic uterine contractions are poor in quality and lack sufficient intensity to dilate and efface the cervix. Contractions of precipitous labor occur rapidly such that labor is completed in less than 3 hours.

21. A nurse is teaching a pregnant woman at risk for preterm labor about what to do if she experiences signs and symptoms. The nurse determines that the teaching was successful when the woman states that if she experiences any symptoms, she will do which of the following? A) "I'll sit down to rest for 30 minutes." B) "I'll try to move my bowels." C) "I'll lie down with my legs raised." D) "I'll drink several glasses of water."

Ans: D Feedback: If the woman experiences any signs and symptoms of preterm labor, she should stop what she is doing and rest for 1 hour, empty her bladder, lie down on her side, drink two to three glasses of water, feel her abdomen and note the hardness of the contraction, and call her health care provider and describe the contraction.

4. When performing newborn resuscitation, which action would the nurse do first? A) Intubate with an appropriate-sized endotracheal tube. B) Give chest compressions at a rate of 80 times per minute. C) Administer epinephrine intravenously. D) Suction the mouth and then the nose.

Ans: D Feedback: After placing the newborn's head in a "sniffing" position, the nurse would suction the mouth and then the nose. This is followed by ventilation, circulation (chest compressions), and administration of epinephrine.

26. A nurse is assisting the anxious parents of a preterm newborn to cope with the situation. Which statement by the nurse would be least appropriate? A) "I'll be here to help you all along the way." B) "What has helped you to deal with stressful situations in the past?" C) "Let me tell you about what you will see when you visit your baby." D) "Forget about what's happened in the past and focus on the now."

Ans: D Feedback: Instead of telling the parents to forget about what's happened, the nurse should review with them the events that have occurred since birth to help them understand and clarify any misconceptions they might have. Other helpful interventions would include telling the parents that the nurse will be with them because this provides them with a physical presence and support; asking about previous coping strategies that worked so that they can use them now; and explaining what is happening and all the equipment being used so they can understand the situation.

4. A newborn has been diagnosed with a Group B streptococcal infection shortly after birth. The nurse understands that the newborn most likely acquired this infection from which of the following? A) Improper handwashing B) Contaminated formula C) Nonsterile catheter insertion D) Mother's birth canal

Ans: D Feedback: Most often, a newborn develops a Group B streptococcus infection during the birthing process when the newborn comes into contact with an infected birth canal. Improper handwashing, contaminated formula, and nonsterile catheter insertion would most likely lead to a late-onset infection, which typically occurs in the nursery due to horizontal transmission.

8. A pregnant client undergoing labor induction is receiving an oxytocin infusion. Which of the following findings would require immediate intervention? A) Fetal heart rate of 150 beats/minute B) Contractions every 2 minutes, lasting 45 seconds C) Uterine resting tone of 14 mm Hg D) Urine output of 20 mL/hour

Ans: D Feedback: Oxytocin can lead to water intoxication. Therefore, a urine output of 20 mL/hour is below acceptable limits of 30 mL/hour and requires intervention. FHR of 150 beats/minute is within the accepted range of 120 to 160 beats/minute. Contractions should occur every 2 to 3 minutes, lasting 40 to 60 seconds. A uterine resting tone greater than 20 mm Hg would require intervention.

1. After spontaneous rupture of membranes, the nurse notices a prolapsed cord. The nurse immediately places the woman in which position? A) Supine B) Side-lying C) Sitting D) Knee-chest

Ans: D Feedback: Pressure on the cord needs to be relieved. Therefore, the nurse would position the woman in a modified Sims, Trendelenburg, or knee-chest position. Supine, side-lying, or sitting would not provide relief of cord compression.

1. The nurse is teaching a group of students about the differences between a full-term newborn and a preterm newborn. The nurse determines that the teaching is effective when the students state that the preterm newborn has: A) Fewer visible blood vessels through the skin B) More subcutaneous fat in the neck and abdomen C) Well-developed flexor muscles in the extremities D) Greater surface area in proportion to weight

Ans: D Feedback: Preterm newborns have large body surface areas compared to weight, which allows an increased transfer of heat from their bodies to the environment. Preterm newborns often have thin transparent skin with numerous visible veins, minimal subcutaneous fat, and poor muscle tone.

22. When assessing a newborn's reflexes, the nurse strokes the newborn's cheek and the newborn turns toward the side that was stroked and begins sucking. The nurse documents which reflex as being positive? A) Palmar grasp reflex B) Tonic neck reflex C) Moro reflex D) Rooting reflex

Ans: D Feedback: The rooting reflex is elicited by stroking the newborn's cheek. The newborn should turn toward the side that was stroked and should begin to make sucking movements. The palmar grasp reflex is elicited by placing a finger on the newborn's open palm. The baby's hand will close around the finger. Attempting to remove the finger causes the grip to tighten. The tonic neck reflex is elicited by having the newborn lie on the back and turning the head to one side. The arm toward which the baby is facing should extend straight away from the body with the hand partially open, whereas the arm on the side away from the face is flexed and the fist is clenched tightly. Reversing the direction to which the face is turned reverses the position. The Moro reflex is elicited by placing the newborn on his or her back, supporting the upper body weight of the supine newborn by the arms using a lifting motion without lifting the newborn off the surface. The arms are released suddenly and the newborn will throw the arms outward and flex the knees and then the arms return to the chest. The fingers also spread to form a C.

1. A woman in labor who received an opioid for pain relief develops respiratory depression. The nurse would expect which agent to be administered? A) Butorphanol B) Fentanyl C) Naloxone D) Promethazine

Ans: C Feedback: Naloxone is an opioid antagonist used to reverse the effects of opioids such as respiratory depression. Butorphanol and fentanyl are opioids and would cause further respiratory depression. Promethazine is an ataractic used as an adjunct to potentiate the effectiveness of the opioid.

13. A nurse is counseling a mother about the immunologic properties of breast milk. The nurse integrates knowledge of immunoglobulins, emphasizing that breast milk is a major source of which immunoglobulin? A) IgA B) IgG C) IgM D) IgE

Ans: A Feedback: A major source of IgA is human breast milk. IgG, found in serum and interstitial fluid, crosses the placenta beginning at approximately 20 to 22 weeks' gestation. IgM is found in blood and lymph fluid and levels are generally low at birth unless there is a congenital intrauterine infection. IgE is not found in breast milk and does not play a major role in defense in the newborn.

8. After the birth of a newborn, which of the following would the nurse do first to assist in thermoregulation? A) Dry the newborn thoroughly. B) Put a hat on the newborn's head. C) Check the newborn's temperature. D) Wrap the newborn in a blanket.

Ans: A Feedback: Drying the newborn immediately after birth using warmed blankets is essential to prevent heat loss through evaporation. Then the nurse would place a cap on the baby's head and wrap the newborn. Assessing the newborn's temperature would occur once these measures were initiated to prevent heat loss.

15. Which of the following is a priority when caring for a woman during the fourth stage of labor? A) Assessing the uterine fundus B) Offering fluids as indicated C) Encouraging the woman to void D) Assisting with perineal care

Ans: A Feedback: During the fourth stage of labor, a priority is to assess the woman's fundus to prevent postpartum hemorrhage. Offering fluids, encouraging voiding, and assisting with perineal care are important but not an immediate priority.

3. A woman has just entered the second stage of labor. The nurse would focus care on which of the following? A) Encouraging the woman to push when she has a strong desire to do so B) Alleviating perineal discomfort with the application of ice packs C) Palpating the woman's fundus for position and firmness D) Completing the identification process of the newborn with the mother

Ans: A Feedback: During the second stage of labor, nursing interventions focus on motivating the woman, encouraging her to put all her efforts toward pushing. Alleviating perineal discomfort with ice packs and palpating the woman's fundus would be appropriate during the fourth stage of labor. Completing the newborn identification process would be appropriate during the third stage of labor.

19. The nurse is reviewing the monitoring strip of a woman in labor who is experiencing a contraction. The nurse notes the time the contraction takes from its onset to reach its highest intensity. The nurse interprets this time as which of the following? A) Increment B) Acme C) Peak D) Decrement

Ans: A Feedback: Each contraction has three phases: increment or the buildup of the contraction; acme or the peak or highest intensity; and the decrement or relaxation of the uterine muscle fibers. The time from the onset to the highest intensity corresponds to the increment.

4. The nurse notes persistent early decelerations on the fetal monitoring strip. Which of the following would the nurse do next? A) Continue to monitor the FHR because this pattern is benign. B) Perform a vaginal exam to assess cervical dilation and effacement. C) Stay with the client while reporting the finding to the physician. D) Administer oxygen after turning the client on her left side.

Ans: A Feedback: Early decelerations are not indicative of fetal distress and do not require intervention. Therefore, the nurse would continue to monitor the fetal heart rate pattern. They are most often seen during the active stage of any normal labor, during pushing, crowning, or vacuum extraction. They are thought to be a result of fetal head compression that results in a reflex vagal response with a resultant slowing of the FHR during uterine contractions. There is no need to perform a vaginal exam, report the finding to the physician, or administer oxygen.

7. A client expresses concern that her 2-hour-old newborn is sleepy and difficult to awaken. The nurse explains that this behavior indicates which of the following? A) Normal progression of behavior B) Probable hypoglycemia C) Physiological abnormality D) Inadequate oxygenation

Ans: A Feedback: From 30 to 120 minutes of age, the newborn enters the second stage of transition, the period of decreased responsiveness or that of sleep or a decrease in activity. More information would be needed to determine if hypoglycemia, a physiologic abnormality, or inadequate oxygenation was present.

11. A woman in labor received an opioid close to the time of birth. The nurse would assess the newborn for which of the following? A) Respiratory depression B) Urinary retention C) Abdominal distention D) Hyperreflexia

Ans: A Feedback: Opioids given close to the time of birth can cause central nervous system depression, including respiratory depression, in the newborn, necessitating the administration of naloxone. Urinary retention may occur in the woman who received neuraxial opioids. Abdominal distention is not associated with opioid administration. Hyporeflexia would be more commonly associated with central nervous system depression due to opioids.

2. Which of the following would indicate to the nurse that the placenta is separating? A) Uterus becomes globular B) Fetal head is at vaginal opening C) Umbilical cord shortens D) Mucous plug is expelled

Ans: A Feedback: Placental separation is indicated by the uterus changing shape to globular and upward rising of the uterus. Additional signs include a sudden trickle of blood from the vaginal opening, and lengthening (not shortening) of the umbilical cord. The fetal head at the vaginal opening is termed crowning and occurs before birth of the head. Expulsion of the mucous plug is a premonitory sign of labor.

7. The fetus of a nulliparous woman is in a shoulder presentation. The nurse would most likely prepare the client for which type of birth? A) Cesarean B) Vaginal C) Forceps-assisted D) Vacuum extraction

Ans: A Feedback: The fetus is in a transverse lie with the shoulder as the presenting part, necessitating a cesarean birth. Vaginal birth, forceps-assisted, and vacuum extraction births are not appropriate.

4. Twenty minutes after birth, a baby begins to move his head from side to side, making eye contact with the mother, and pushes his tongue out several times. The nurse interprets this as indicating which of the following? A) A good time to initiate breast-feeding B) The period of decreased responsiveness preceding sleep C) The need to be alert for gagging and vomiting D) Evidence that the newborn is becoming chilled

Ans: A Feedback: The newborn is demonstrating behaviors indicating the first period of reactivity, which usually begins at birth and lasts for the first 30 minutes. This is a good time to initiate breast-feeding. Decreased responsiveness occurs from 30 to 120 minutes of age and is characterized by muscle relaxation and diminished responsiveness to outside stimuli. There is no indication that the newborn may experience gagging or vomiting. Chilling would be evidenced by tachypnea, decreased activity, and hypotonia.

19. After teaching a group of students about fetal heart rate patterns, the instructor determines the need for additional teaching when the students identify which of the following as indicating normal fetal acid-base status? (Select all that apply.) A) Sinusoidal pattern B) Recurrent variable decelerations C) Fetal bradycardia D) Absence of late decelerations E) Moderate baseline variability

Ans: A, B, C Feedback: Predictors of normal fetal acid-base status include a baseline rate between 110 and 160 bpm, moderate baseline variability, and absences of later or variable decelerations. Sinusoidal pattern, recurrent variable decelerations, and fetal bradycardia are predictive of abnormal fetal acid-base status.

17. After teaching a group of nursing students about variations in newborn head size and appearance, the instructor determines that the teaching was successful when the students identify which of the following as a normal variation? (Select all that apply.) A) Cephalhematoma B) Molding C) Closed fontanels D) Caput succedaneum E) Posterior fontanel diameter 1.5 cm

Ans: A, B, D Feedback: Normal variations in newborn head size and appearance include cephalhematoma, molding, and caput succedaneum. Microcephaly, closed fontanels, or a posterior fontanel diameter greater than 1 cm are considered abnormal.

18. A nurse is preparing a class for pregnant women about labor and birth. When describing the typical movements that the fetus goes through as it travels through the passageway, which of the following would the nurse most likely include? (Select all that apply.) A) Internal rotation B) Abduction C) Descent D) Pronation E) Flexion

Ans: A, C, E Feedback: The positional changes that occur as the fetus moves through the passageway are called the cardinal movements of labor and include engagement, descent, flexion, internal rotation, extension, external rotation, and expulsion. The fetus does not undergo abduction or pronation.

8. A woman in labor is to receive continuous internal electronic fetal monitoring. The nurse reviews the woman's medical record to ensure which of the following as being required? A) Intact membranes B) Cervical dilation of 2 cm or more C) Floating presenting fetal part D) A neonatologist to insert the electrode

Ans: B Feedback: For continuous internal electronic fetal monitoring, four criteria must be met: ruptured membranes, cervical dilation of at least 2 cm, fetal presenting part low enough to allow placement of the electrode, and a skilled practitioner available to insert the electrode.

13. Assessment of a newborn reveals uneven gluteal (buttocks) skin creases and a "clunk" when Ortolani's maneuver is performed. Which of the following would the nurse suspect? A) Slipping of the periosteal joint B) Developmental hip dysplasia C) Normal newborn variation D) Overriding of the pelvic bone

Ans: B Feedback: A "clunk" indicates the femoral head hitting the acetabulum as the head reenters the area. This, along with uneven gluteal creases, suggests developmental hip dysplasia. These findings are not a normal variation and are not associated with slipping of the periosteal joint or overriding of the pelvic bone.

5. Just after delivery, a newborn's axillary temperature is 94° C. What action would be most appropriate? A) Assess the newborn's gestational age. B) Rewarm the newborn gradually. C) Observe the newborn every hour. D) Notify the physician if the temperature goes lower.

Ans: B Feedback: A newborn's temperature is typically maintained at 36.5 to 37.5° C (97.7 to 99.7° F). Since this newborn's temperature is significantly lower, the nurse should institute measures to rewarm the newborn gradually. Assessment of gestational age is completed regardless of the newborn's temperature. Observation would be inappropriate because lack of action may lead to a further lowering of the temperature. The nurse should notify the physician of the newborn's current temperature since it is outside normal parameters.

3. When assessing cervical effacement of a client in labor, the nurse assesses which of the following characteristics? A) Extent of opening to its widest diameter B) Degree of thinning C) Passage of the mucous plug D) Fetal presenting part

Ans: B Feedback: Effacement refers to the degree of thinning of the cervix. Cervical dilation refers to the extent of opening at the widest diameter. Passage of the mucous plug occurs with bloody show is a premonitory sign of labor. The fetal presenting part is determined by vaginal examination and is commonly the head (cephalic), pelvis (breech), or shoulder.

14. A woman in her third trimester comes to the clinic for a prenatal visit. During assessment the woman reports that her breathing has become much easier in the last week but she has noticed increased pelvic pressure, cramping, and lower back pain. The nurse determines that which of the following has most likely occurred? A) Cervical dilation B) Lightening C) Bloody show D) Braxton-Hicks contractions

Ans: B Feedback: Lightening occurs when the fetal presenting part begins to descend into the maternal pelvis. The uterus lowers and moves into the maternal pelvis. The shape of the abdomen changes as a result of the change in the uterus. The woman usually notes that her breathing is much easier. However, she may complain of increased pelvic pressure, cramping, and lower back pain. Although cervical dilation also may be occurring, it does not account for the woman's complaints. Bloody show refers to passage of the mucous plug that fills the cervical canal during pregnancy. It occurs with the onset of labor. Braxton-Hicks contractions increase in strength and frequency and aid in moving the cervix from a posterior position to an anterior position. They also help in ripening and softening the cervix.

4. A woman calls the health care facility stating that she is in labor. The nurse would urge the client to come to the facility if the client reports which of the following? A) Increased energy level with alternating strong and weak contractions B) Moderately strong contractions every 4 minutes, lasting about 1 minute C) Contractions noted in the front of abdomen that stop when she walks D) Pink-tinged vaginal secretions and irregular contractions lasting about 30 seconds

Ans: B Feedback: Moderately strong regular contractions 60 seconds in duration indicate that the client is probably in the active phase of the first stage of labor. Alternating strong and weak contractions, contractions in the front of the abdomen that change with activity, and pink-tinged secretions with irregular contractions suggest false labor.

9. A client is admitted to the labor and birthing suite in early labor. On review of her medical record, the nurse determines that the client's pelvic shape as identified in the antepartal progress notes is the most favorable one for a vaginal delivery. Which pelvic shape would the nurse have noted? A) Platypelloid B) Gynecoid C) Android D) Anthropoid

Ans: B Feedback: The most favorable pelvic shape for vaginal delivery is the gynecoid shape. The anthropoid pelvis is favorable for vaginal birth but it is not the most favorable shape. The android pelvis is not considered favorable for a vaginal birth because descent of the fetal head is slow and failure of the fetus to rotate is common. Women with a platypelloid pelvis usually require cesarean birth.

6. A new mother is changing the diaper of her 20-hour-old newborn and asks why the stool is almost black. Which response by the nurse would be most appropriate? A) "You probably took iron during your pregnancy." B) "This is meconium stool, normal for a newborn." C) "I'll take a sample and check it for possible bleeding." D) "This is unusual and I need to report this."

Ans: B Feedback: The mother is describing meconium. Meconium is greenish-black and tarry and usually passed within 12 to 24 hours of birth. This is a normal finding. Iron can cause stool to turn black, but this would not be the case here. The stool is a normal occurrence and does not need to be checked for blood or reported.

26. After teaching a group of nursing students about a neutral thermal environment, the instructor determines that the teaching was successful when the students identify which of the following as the newborn's primary method of heat production? A) Convection B) Nonshivering thermogenesis C) Cold stress D) Bilirubin conjugation

Ans: B Feedback: The newborn's primary method of heat production is through nonshivering thermogenesis, a process in which brown fat (adipose tissue) is oxidized in response to cold exposure. Convection is a mechanism of heat loss. Cold stress results with excessive heat loss that requires the newborn to use compensatory mechanisms to maintain core body temperature. Bilirubin conjugation is a mechanism by which bilirubin in the blood is eliminated.

25. A nursing student is preparing a presentation on minimizing heat loss in the newborn. Which of the following would the student include as a measure to prevent heat loss through convection? A) Placing a cap on a newborn's head B) Working inside an isolette as much as possible. C) Placing the newborn skin-to-skin with the mother D) Using a radiant warmer to transport a newborn

Ans: B Feedback: To prevent heat loss by convection, the nurse would keep the newborn out of direct cool drafts (open doors, windows, fans, air conditioners) in the environment, work inside an isolette as much as possible and minimize opening portholes that allow cold air to flow inside, and warm any oxygen or humidified air that comes in contact with the newborn. Placing a cap on the newborn's head would help minimize heat loss through evaporation. Placing the newborn skin-to-skin with the mother helps to prevent heat loss through conduction. Using a radiant warmer to transport a newborn helps minimize heat loss through radiation.

19. A nurse is assessing a newborn who is about 41/2 hours old. The nurse would expect this newborn to exhibit which of the following? (Select all that apply.) A) Sleeping B) Interest in environmental stimuli C) Passage of meconium D) Difficulty arousing the newborn E) Spontaneous Moro reflexes

Ans: B, C Feedback: The newborn is in the second period of reactivity, which begins as the newborn awakens and shows an interest in environmental stimuli. This period lasts 2 to 8 hours in the normal newborn (Boxwell, 2010). Heart and respiratory rates increase. Peristalsis also increases. Thus, it is not uncommon for the newborn to pass meconium or void during this period. In addition, motor activity and muscle tone increase in conjunction with an increase in muscular coordination. Spontaneous Moro reflexes are noted during the first period of reactivity. Sleeping and difficulty arousing the newborn reflect the period of decreased responsiveness.

25. A pregnant woman admitted to the labor and birth suite undergoes rapid HIV testing and is found to be HIV-positive. Which of the following would the nurse expect to include when developing a plan of care for this women? (Select all that apply.) A) Administration of penicillin G at the onset of labor B) Avoidance of scalp electrodes for fetal monitoring C) Refraining from obtaining fetal scalp blood for pH testing D) Administering zidovudine at the onset of labor. E) Electing for the use of forceps-assisted delivery

Ans: B, C, D Feedback: To reduce perinatal transmission, HIV-positive women are given zidovudine (ZDV) (2 mg/kg IV over an hour, and then a maintenance infusion of 1 mg/kg per hour until birth) or a single 200-mg oral dose of nevirapine at the onset of labor; the newborn is given ZDV orally (2 mg/kg body weight every 6 hours) and should be continued for 6 weeks (Gardner, Carter, Enzman-Hines, & Hernandez, 2011). To further reduce the risk of perinatal transmission, ACOG and the U.S. Public Health Service recommend that HIV-infected women with plasma viral loads of more than 1,000 copies per milliliter be counseled regarding the benefits of elective cesarean birth (Reshi & Lone, 2010). Additional interventions to reduce the transmission risk would include avoiding use of scalp electrode for fetal monitoring or doing a scalp blood sampling for fetal pH, delaying amniotomy, encouraging formula feeding after birth, and avoiding invasive procedures such as forceps or vacuum-assisted devices.

24. A nurse is completing the assessment of a woman admitted to the labor and birth suite. Which of the following would the nurse expect to include as part of the physical assessment? (Select all that apply.) A) Current pregnancy history B) Fundal height measurement C) Support system D) Estimated date of birth E) Membrane status F) Contraction pattern

Ans: B, E, F Feedback: As part of the admission physical assessment, the nurse would assess fundal height, membrane status and contractions. Current pregnancy history, support systems, and estimated date of birth would be obtained when collecting the maternal health history.

16. When palpating the fundus during a contraction, the nurse notes that it feels like a chin. The nurse interprets this finding as indicating which type of contraction? A) Intense B) Strong C) Moderate D) Mild

Ans: C Feedback: A contraction that feels like the chin typically represents a moderate contraction. A contraction described as feeling like the tip of the nose indicates a mild contraction. A strong contraction feels like the forehead.

1. When explaining how a newborn adapts to extrauterine life, the nurse would describe which body systems as undergoing the most rapid changes? A) Gastrointestinal and hepatic B) Urinary and hematologic C) Respiratory and cardiovascular D) Neurological and integumentary

Ans: C Feedback: Although all the body systems of the newborn undergo changes, respiratory gas exchange along with circulatory modifications must occur immediately to sustain extrauterine life.

2. The nurse encourages the mother of a healthy newborn to put the newborn to the breast immediately after birth for which reason? A) To aid in maturing the newborn's sucking reflex B) To encourage the development of maternal antibodies C) To facilitate maternal-infant bonding D) To enhance the clearing of the newborn's respiratory passages

Ans: C Feedback: Breast-feeding can be initiated immediately after birth. This immediate mother-newborn contact takes advantage of the newborn's natural alertness and fosters bonding. This contact also reduces maternal bleeding and stabilizes the newborn's temperature, blood glucose level, and respiratory rate. It is not associated with maturing the sucking reflex, encouraging the development of maternal antibodies, or aiding in clearing of the newborn's respiratory passages.

16. A nurse is documenting fetal lie of a woman in labor. Which term would the nurse most likely use? A) Flexion B) Extension C) Longitudinal D) Cephalic

Ans: C Feedback: Fetal lie refers to the relationships of the long axis (spine) of the fetus to the long axis (spine) of the mother. There are two primary lies: longitudinal and transverse. Flexion and extension are terms used to describe fetal attitude. Cephalic is a term used to describe fetal presentation.

29. A nurse is assisting with the delivery of a newborn. The fetal head has just emerged. Which of the following would be done next? A) Suctioning of the mouth and nose B) Clamping of the umbilical cord C) Checking for the cord around the neck D) Drying of the newborn

Ans: C Feedback: Once the fetal head has emerged, the primary care provider explores the fetal neck to see if the umbilical cord is wrapped around it. If it is, the cord is slipped over the head to facilitate delivery. Then the health care provider suctions the newborn's mouth first (because the newborn is an obligate nose breather) and then the nares with a bulb syringe to prevent aspiration of mucus, amniotic fluid, or meconium. Finally the umbilical cord is double-clamped and cut between the clamps. The newborn is placed under the radiant warmer, dried, assessed, wrapped in warm blankets and placed on the woman's abdomen for warmth and closeness.

12. When applying the ultrasound transducers for continuous external electronic fetal monitoring, at which location would the nurse place the transducer to record the FHR? A) Over the uterine fundus where contractions are most intense B) Above the umbilicus toward the right side of the diaphragm C) Between the umbilicus and the symphysis pubis D) Between the xiphoid process and umbilicus

Ans: C Feedback: The ultrasound transducer is positioned on the maternal abdomen in the midline between the umbilicus and the symphysis pubis. The tocotransducer is placed over the uterine fundus in the area of greatest contractility.

5. A woman is in the first stage of labor. The nurse would encourage her to assume which position to facilitate the progress of labor? A) Supine B) Lithotomy C) Upright D) Knee-chest

Ans: C Feedback: The use of any upright position helps to reduce the length of labor. Research validates that nonmoving back-lying positions such as supine and lithotomy positions during labor are not healthy. The knee-chest position would assist in rotating the fetus in a posterior position.

21. A nurse is explaining the use of therapeutic touch as a pain relief measure during labor. Which of the following would the nurse include in the explanation? A) "This technique focuses on manipulating body tissues." B) "The technique requires focusing on a specific stimulus." C) "This technique redirects energy fields that lead to pain." D) "The technique involves light stroking of the abdomen with breathing."

Ans: C Feedback: Therapeutic touch is an energy therapy and is based on the premise that the body contains energy fields that lead to either good or ill health and that the hands can be used to redirect the energy fields that lead to pain. Attention focusing and imagery involve focusing on a specific stimulus. Massage focuses on manipulating body tissues. Effleurage involves light stroking of the abdomen in rhythm with breathing.

20. A nurse is assessing a woman in labor. Which finding would the nurse identify as a cause for concern during a contraction? A) Heart rate increase from 76 bpm to 90 bpm B) Blood pressure rise from 110/60 mm Hg to 120/74 C) White blood cell count of 12,000 cells/mm3 D) Respiratory rate of 10 breaths /minute

Ans: D Feedback: During labor, the mother experiences various physiologic responses including an increase in heart rate by 10 to 20 bpm, a rise in blood pressure by up to 35 mm Hg during a contraction, an increase in white blood cell count to 25,000 to 30,000 cells/mm3, perhaps as a result of tissue trauma, and an increase in respiratory rate with greater oxygen consumption due to the increase in metabolism. A drop in respiratory rate would be a cause for concern.

16. The nurse is assessing the respirations of several newborns. The nurse would notify the health care provider for the newborn with which respiratory rate at rest? A) 38 breaths per minute B) 46 breaths per minute C) 54 breaths per minute D) 68 breaths per minute

Ans: D Feedback: After respirations are established in the newborn, they are shallow and irregular, ranging from 30 to 60 breaths per minute, with short periods of apnea (less than 15 seconds). Thus a newborn with a respiratory rate below 30 or above 60 breaths per minute would require further evaluation.

7. A client states, "I think my waters broke! I felt this gush of fluid between my legs." The nurse tests the fluid with Nitrazine paper and confirms membrane rupture if the paper turns: A) Yellow B) Olive green C) Pink D) Blue

Ans: D Feedback: Amniotic fluid is alkaline and turns Nitrazine paper blue. Nitrazine paper that remains yellow to olive green suggests that the membranes are most likely intact.

10. A woman in labor has chosen to use hydrotherapy as a method of pain relief. Which statement by the woman would lead the nurse to suspect that the woman needs additional teaching? A) "The warmth and buoyancy of the water has a nice relaxing effect." B) "I can stay in the bath for as long as I feel comfortable." C) "My cervix should be dilated more than 5 cm before I try using this method." D) "The temperature of the water should be at least 105°F."

Ans: D Feedback: Hydrotherapy is an effective pain relief method. The water temperature should not exceed body temperature. Therefore, a temperature of 105° F would be too warm. The warmth and buoyancy have a relaxing effect and women are encouraged to stay in the bath as long as they feel comfortable. The woman should be in active labor with cervical dilation greater than 5 cm.

22. A nurse is caring for several women in labor. The nurse determines that which woman is in the transition phase of labor? A) Contractions every 5 minutes, cervical dilation 3 cm B) Contractions every 3 minutes, cervical dilation 5 cm C) Contractions every 21/2 minutes, cervical dilation 7 cm D) Contractions every 1 minute, cervical dilation 9 cm

Ans: D Feedback: The transition phase is characterized by strong contractions occurring every 1 to 2 minutes and cervical dilation from 8 to 10 cm. Contractions every 5 minutes with cervical dilation of 3 cm is typical of the latent phase. Contractions every 3 minutes with cervical dilation of 5 cm and contractions every 21/2 minutes with cervical dilation of 7 cm suggest the active phase of labor.

10. After teaching new parents about the sensory capabilities of their newborn, the nurse determines that the teaching was successful when they identify which sense as being the least mature? A) Hearing B) Touch C) Taste D) Vision

Ans: D Feedback: Vision is the least mature sense at birth. Hearing is well developed at birth, evidenced by the newborn's response to noise by turning. Touch is evidenced by the newborn's ability to respond to tactile stimuli and pain. A newborn can distinguish between sweet and sour by 72 hours of age.

13. Assessment of a fetus identifies the buttocks as the presenting part, with the legs extended upward. The nurse identifies this as which type of breech presentation? A) Frank B) Full C) Complete D) Footling

Ans: A Feedback: In a frank breech, the buttocks present first, with both legs extended up toward the face. In a full or complete breech, the fetus sits cross-legged above the cervix. In a footling breech, one or both legs are presenting.

24. During a follow-up prenatal visit, a pregnant woman asks the nurse, "How long do you think I will be in labor?" Which response by the nurse would be most appropriate? A) "It's difficult to predict how your labor will progress, but we'll be there for you the entire time." B) "Since this is your first pregnancy, you can estimate it will be about 10 hours." C) "It will depend on how big the baby is when you go into labor." D) "Time isn't important; your health and the baby's health are key."

Ans: A Feedback: It is difficult to predict how a labor will progress and therefore equally difficult to determine how long a woman's labor will last. There is no way to estimate the likely strength and frequency of uterine contractions, the extent to which the cervix will soften and dilate, and how much the fetal head will mold to fit the birth canal. We cannot know beforehand whether the complex fetal rotations needed for an efficient labor will take place properly. All of these factors are unknowns when a woman starts labor. Telling the woman an approximate time would be inappropriate because there is no way to determine the length of labor. It is highly individualized. Although fetal size and maternal and fetal health are important considerations, these responses do not address the woman's concern.

15. A nurse is developing a teaching plan for the parents of a newborn. When describing the neurologic development of a newborn to his parents, the nurse would explain that the development occurs in which fashion? A) Head-to-toe B) Lateral-to-medial C) Outward-to-inward D) Distal-to-caudal

Ans: A Feedback: Neurologic development follows a cephalocaudal (head-to-toe) and proximal-distal (center to outside) pattern.

17. A new mother asks the nurse, "Why has my baby lost weight since he was born?" The nurse integrates knowledge of which of the following when responding to the new mother? A) Insufficient calorie intake B) Shift of water from extracellular space to intracellular space C) Increase in stool passage D) Overproduction of bilirubin

Ans: A Feedback: Normally, term newborns lose 5% to 10% of their birth weight as a result of insufficient caloric intake within the first week after birth, shifting of intracellular water to extracellular space, and insensible water loss. Stool passage and bilirubin have no effect on weight loss.

25. A nurse is describing how the fetus moves through the birth canal. Which of the following would the nurse identify as being most important in allowing the fetal head to move through the pelvis? A) Sutures B) Fontanelles C) Frontal bones D) Biparietal diameter

Ans: A Feedback: Sutures are important because they allow the cranial bones to overlap in order for the head to adjust in shape (elongate) when pressure is exerted on it by uterine contractions or the maternal bony pelvis. Fontanelles are the intersections formed by the sutures. The frontal bones, along with the parietal and occipital bones are bones of the cranium that are soft and pliable. The biparietal diameter is an important diameter that can affect the birth process.

14. The nurse strokes the lateral sole of the newborn's foot from the heel to the ball of the foot when evaluating which reflex? A) Babinski B) Tonic neck C) Stepping D) Plantar grasp

Ans: A Feedback: The Babinski reflex is elicited by stroking the lateral sole of the newborn's foot from the heel toward and across the ball of the foot. The tonic neck reflex is tested by having the newborn lie on his back and then turn his head to one side. The stepping reflex is elicited by holding the newborn upright and inclined forward with the soles of the feet on a flat surface. The plantar grasp reflex is elicited by placing a finger against the area just below the newborn's toes.

6. The nurse is performing Leopold's maneuvers to determine fetal presentation, position, and lie. Which action would the nurse do first? A) Feel for the fetal buttocks or head while palpating the abdomen. B) Feel for the fetal back and limbs as the hands move laterally on the abdomen. C) Palpate for the presenting part in the area just above the symphysis pubis. D) Determine flexion by pressing downward toward the symphysis pubis.

Ans: A Feedback: The first maneuver involves feeling for the buttocks and head. Next the nurse palpates on which side the fetal back is located. The third maneuver determines presentation and involves palpating the area just above the symphysis pubis. The final maneuver determines attitude and involves applying downward pressure in the direction of the symphysis pubis.

8. Assessment of a woman in labor reveals cervical dilation of 3 cm, cervical effacement of 30%, and contractions occurring every 7 to 8 minutes, lasting about 40 seconds. The nurse determines that this client is in: A) Latent phase of the first stage B) Active phase of the first stage C) Transition phase of the first stage D) Perineal phase of the second stage

Ans: A Feedback: The latent phase of the first stage of labor involves cervical dilation of 0 to 3 cm, cervical effacement of 0% to 40%, and contractions every 5 to 10 minutes lasting 30 to 45 seconds. The active phase is characterized by cervical dilation of 4 to 7 cm, effacement of 40% to 80%, and contractions occurring every 2 to 5 minutes lasting 45 to 60 seconds. The transition phase is characterized by cervical dilation of 8 to 10 cm, effacement of 80% to 100%, and contractions occurring every 1 to 2 minutes lasting 60 to 90 seconds. The perineal phase of the second stage occurs with complete cervical dilation and effacement, contractions occurring every 2 to 3 minutes and lasting 60 to 90 seconds, and a tremendous urge to push by the mother.

7. When assessing a newborn 1 hour after birth, the nurse measures an axillary temperature of 95.8° F, an apical pulse of 114 beats/minute, and a respiratory rate of 60 breaths/minute. Which nursing diagnosis takes highest priority? A) Hypothermia related to heat loss during birthing process B) Impaired parenting related to addition of new family member C) Risk for deficient fluid volume related to insensible fluid loss D) Risk for infection related to transition to extrauterine environment

Ans: A Feedback: The newborn's heart rate is slightly below the accepted range of 120 to 160 beats/minute; the respiratory rate is at the high end of the accepted range of 30 to 60 breaths per minute. However, the newborn's temperature is significantly below the accepted range of 97.7 to 99.5° F. Therefore, the priority nursing diagnosis is hypothermia. There is no information to suggest impaired parenting. Additional information is needed to determine if there is a risk for deficient fluid volume or a risk for infection.

4. Assessment of a newborn reveals a heart rate of 180 beats/minute. To determine whether this finding is a common variation rather than a sign of distress, what else does the nurse need to know? A) How many hours old is this newborn? B) How long ago did this newborn eat? C) What was the newborn's birth weight? D) Is acrocyanosis present?

Ans: A Feedback: The typical heart rate of a newborn ranges from 120 to 160 beats per minute with wide fluctuation during activity and sleep. Typically heart rate is assessed every 30 minutes until stable for 2 hours after birth. The time of the newborn's last feeding and his birth weight would have no effect on his heart rate. Acrocyanosis is a common normal finding in newborns.

17. A nurse palpates a woman's fundus to determine contraction intensity. Which of the following would be most appropriate for the nurse to use for palpation? A) Finger pads B) Palm of the hand C) Finger tips D) Back of the hand

Ans: A Feedback: To palpate the fundus for contraction intensity, the nurse would place the pads of the fingers on the fundus and describe how it feels. Using the finger tips, palm, or back of the hand would be inappropriate.

1. Prior to discharging a 24-hour-old newborn, the nurse assesses her respiratory status. Which of the following would the nurse expect to assess? A) Respiratory rate 45, irregular B) Costal breathing pattern C) Nasal flaring, rate 65 D) Crackles on auscultation

Ans: A Feedback: Typically, respirations in a 24-hour-old newborn are symmetric, slightly irregular, shallow, and unlabored at a rate of 30 to 60 breaths/minute. The breathing pattern is primarily diaphragmatic. Nasal flaring, rates above 60 breaths per minute, and crackles suggest a problem.

23. A nurse is describing the different types of regional analgesia and anesthesia for labor to a group of pregnant women. Which statement by the group indicates that the teaching was successful? A) "We can get up and walk around after receiving combined spinal-epidural analgesia." B) "Higher anesthetic doses are needed for patient-controlled epidural analgesia." C) "A pudendal nerve block is highly effective for pain relief in the first stage of labor." D) "Local infiltration using lidocaine is an appropriate method for controlling contraction pain."

Ans: A Feedback: When compared with traditional epidural or spinal analgesia, which often keeps the woman lying in bed, combined spinal-epidural analgesia allows the woman to ambulate ("walking epidural"). Patient-controlled epidural analgesia provides equivalent analgesia with lower anesthetic use, lower rates of supplementation, and higher client satisfaction. Pudendal nerve blocks are used for the second stage of labor, an episiotomy, or an operative vaginal birth with outlet forceps or vacuum extractor. Local infiltration using lidocaine does not alter the pain of uterine contractions, but it does numb the immediate area of the episiotomy or laceration.

2. A client's membranes spontaneously ruptured, as evidenced by a gush of clear fluid with a contraction. Which of the following would the nurse do next? A) Check the fetal heart rate. B) Perform a vaginal exam. C) Notify the physician immediately. D) Change the linen saver pad.

Ans: A Feedback: When membranes rupture, the priority focus is on assessing fetal heart rate first to identify a deceleration, which might indicate cord compression secondary to cord prolapse. A vaginal exam may be done later to evaluate for continued progression of labor. The physician should be notified, but this is not a priority at this time. Changing the linen saver pad would be appropriate once the fetal status is determined and the physician has been notified.

26. Which position would be most appropriate for the nurse to suggest as a comfort measure to a woman who is in the first stage of labor? (Select all that apply.) A) Walking with partner support B) Straddling with forward leaning over a chair C) Closed knee-chest position D) Rocking back and forth with foot on chair E) Supine with legs raised at a 90-degree angle

Ans: A, B, D Feedback: Positioning during the first stage of labor includes walking with support from the partner, side-lying with pillows between the knees, leaning forward by straddling a chair, table, or bed or kneeling over a birthing ball, lunging by rocking weight back and forth with a foot up on a chair or birthing ball or an open knee-chest position.

15. After teaching a group of students about the factors affecting the labor process, the instructor determines that the teaching was successful when the group identifies which of the following as a component of the true pelvis? (Select all that apply.) A) Pelvic inlet B) Cervix C) Mid pelvis D) Pelvic outlet E) Vagina F) Pelvic floor muscles

Ans: A, C, D Feedback: The true pelvis is made up of three planes: the pelvic inlet, mid pelvis, and pelvic outlet. The cervix, vagina, and pelvic floor muscles are the soft tissues of the passageway.

24. A nursing instructor is preparing a class on newborn adaptations. When describing the change from fetal to newborn circulation, which of the following would the instructor most likely include? (Select all that apply.) A) Decrease in right atrial pressure leads to closure of the foramen ovale. B) Increase in oxygen levels leads to a decrease in systemic vascular resistance. C) Onset of respirations leads to a decrease in pulmonary vascular resistance. D) Increase in pressure in the left atrium results from increases in pulmonary blood flow. E) Closure of the ductus venosus eventually forces closure of the ductus arteriosus.

Ans: A, C, D, E Feedback: When the umbilical cord is clamped, the first breath is taken and the lungs begin to function. As a result, systemic vascular resistance increases and blood return to the heart via the inferior vena cava decreases. Concurrently with these changes, there is a rapid decrease in pulmonary vascular resistance and an increase in pulmonary blood flow (Boxwell, 2010). The foramen ovale functionally closes with a decrease in pulmonary vascular resistance, which leads to a decrease in right-sided heart pressures. An increase in systemic pressure, after clamping of the cord, leads to an increase in left-sided heart pressures. Ductus arteriosus, ductus venosus, and umbilical vessels that were vital during fetal life are no longer needed.

21. A newborn is experiencing cold stress. Which of the following would the nurse expect to assess? (Select all that apply.) A) Respiratory distress B) Decreased oxygen needs C) Hypoglycemia D) Metabolic alkalosis E) Jaundice

Ans: A, C, E Feedback: Cold stress in the newborn can lead to the following problems if not reversed: depleted brown fat stores, increased oxygen needs, respiratory distress, increased glucose consumption leading to hypoglycemia, metabolic acidosis, jaundice, hypoxia, and decreased surfactant production.

23. A nurse is preparing a presentation for a group of pregnant women about the labor experience. Which of the following would the nurse most likely include when discussing measures to promote coping for a positive labor experience? (Select all that apply.) A) Presence of a support partner B) View of birth as a stressor C) Low anxiety level D) Fear of loss of control E) Participation in a pregnancy exercise program

Ans: A, C, E Feedback: Numerous factors can affect a woman's coping ability during labor and birth. Having the presence and support of a valued partner during labor, engaging in exercise during pregnancy, viewing the birthing experience as a meaningful rather than stressful event, and a low anxiety level can promote a woman's ability to cope. Excessive anxiety may interfere with the labor progress, and fear of labor and loss of control may enhance pain perception, increasing the fear.

18. The nurse observes the stool of a newborn who has begun to breast-feed. Which of the following would the nurse expect to find? A) Greenish black, tarry stool B) Yellowish-brown, seedy stool C) Yellow-gold, stringy stool D) Yellowish-green, pasty stool

Ans: B Feedback: After feedings are initiated, a transitional stool develops, which is greenish brown to yellowish brown, thinner in consistency, and seedy in appearance. Meconium stool is greenish black and tarry. The last development in the stool pattern is the milk stool. Milk stools of the breast-fed newborn are yellow-gold, loose, and stringy to pasty in consistency, and typically sour-smelling. The milk stools of the formula-fed newborn vary depending on the type of formula ingested. They may be yellow, yellow-green, or greenish and loose, pasty, or formed in consistency, and they have an unpleasant odor.

6. The parents of a newborn become concerned when they notice that their baby seems to stop breathing for a few seconds. After confirming the parents' findings by observing the newborn, which of the following actions would be most appropriate? A) Notify the health care provider immediately. B) Assess the newborn for signs of respiratory distress. C) Reassure the parents that this is an expected pattern. D) Tell the parents not to worry since his color is fine.

Ans: B Feedback: Although periods of apnea of less than 20 seconds can occur, the nurse needs to gather additional information about the newborn's respiratory status to determine if this finding is indicative of a developing problem. Therefore, the nurse would need to assess for signs of respiratory distress. Once this information is obtained, then the nurse can notify the health care provider or explain that this finding is an expected one. However, it would be inappropriate to tell the parents not to worry, because additional information is needed. Also, telling them not to worry ignores their feelings and is not therapeutic.

18. A woman's amniotic fluid is noted to be cloudy. The nurse interprets this finding as which of the following? A) Normal B) Possible infection C) Meconium passage D) Transient fetal hypoxia

Ans: B Feedback: Amniotic fluid should be clear when the membranes rupture, either spontaneously or artificially through an amniotomy (a disposable plastic hook [Amnihook] is used to perforate the amniotic sac). Cloudy or foul-smelling amniotic fluid indicates infection. Green fluid may indicate that the fetus has passed meconium secondary to transient hypoxia, prolonged pregnancy, cord compression, intrauterine growth restriction, maternal hypertension, diabetes, or chorioamnionitis; however, it is considered a normal occurrence if the fetus is in a breech presentation.

10. During a physical assessment of a newborn, the nurse observes bluish markings across the newborn's lower back. The nurse documents this finding as which of the following? A) Milia B) Mongolian spots C) Stork bites D) Birth trauma

Ans: B Feedback: Mongolian spots are blue or purple splotches that appear on the lower back and buttocks of newborns. Milia are unopened sebaceous glands frequently found on a newborn's nose. Stork bites are superficial vascular areas found on the nape of the neck and eyelids and between the eyes and upper lip. Birth trauma would be manifested by bruising, swelling, and possible deformity.

9. Assessment of a newborn reveals rhythmic spontaneous movements. The nurse interprets this as indicating: A) Habituation B) Motor maturity C) Orientation D) Social behaviors

Ans: B Feedback: Motor maturity is evidenced by rhythmic, spontaneous movements. Habituation is manifested by the newborn's ability to respond to the environment appropriately. Orientation involves the newborn's response to new stimuli, such as turning the head to a sound. Social behaviors involve cuddling and snuggling into the arms of a parent.

23. A nurse is reviewing the laboratory test results of a newborn. Which result would the nurse identify as a cause for concern? A) Hemoglobin 19 g/dL B) Platelets 75,000/uL C) White blood cells 20,000/mm3 D) Hematocrit 52%

Ans: B Feedback: Normal newborn platelets range from 150,00 to 350,000/uL. Normal hemoglobin ranges from 17 to 23g/dL, and normal hematocrit ranges from 46% to 68%. Normal white blood cell count ranges from 10,000 to 30,000/mm3.

28. A nurse is assessing a woman after birth and notes a second-degree laceration. The nurse interprets this as indicating that the tear extends through which of the following? A) Skin B) Muscles of perineal body C) Anal sphincter D) Anterior rectal wall

Ans: B Feedback: The extent of the laceration is defined by depth: a first-degree laceration extends through the skin; a second-degree laceration extends through the muscles of the perineal body; a third-degree laceration continues through the anal sphincter muscle; and a fourth-degree laceration also involves the anterior rectal wall.

11. After teaching a group of students about the maternal bony pelvis, which statement by the group indicates that the teaching was successful? A) The bony pelvis plays a lesser role during labor than soft tissue. B) The pelvic outlet is associated with the true pelvis. C) The false pelvis lies below the imaginary linea terminalis. D) The false pelvis is the passageway through which the fetus travels.

Ans: B Feedback: The maternal bony pelvis consists of the true and false portions. The true pelvis is made up of three planes—the inlet, the mid pelvis, and the outlet. The bony pelvis is the more important part of the passageway because it is relatively unyielding. The false pelvis lies above the imaginary linea terminalis. The true pelvis is the bony passageway through which the fetus must travel.

3. When making a home visit, the nurse observes a newborn sleeping on his back in a bassinet. In one corner of the bassinet is a soft stuffed animal and at the other end is a bulb syringe. The nurse determines that the mother needs additional teaching because of which of the following? A) The newborn should not be sleeping on his back. B) Stuffed animals should not be in areas where infants sleep. C) The bulb syringe should not be kept in the bassinet. D) This newborn should be sleeping in a crib.

Ans: B Feedback: The nurse should instruct the mother to remove all fluffy bedding, quilts, stuffed animals, and pillows from the crib to prevent suffocation. Newborns and infants should be placed on their backs to sleep. Having the bulb syringe nearby in the bassinet is appropriate. Although a crib is the safest sleeping location, a bassinet is appropriate initially.

21. When describing the stages of labor to a pregnant woman, which of the following would the nurse identify as the major change occurring during the first stage? A) Regular contractions B) Cervical dilation C) Fetal movement through the birth canal D) Placental separation

Ans: B Feedback: The primary change occurring during the first stage of labor is progressive cervical dilation. Contractions occur during the first and second stages of labor. Fetal movement through the birth canal is the major change during the second stage of labor. Placental separation occurs during the third stage of labor.

3. After teaching a class about hepatic system adaptations after birth, the instructor determines that the teaching was successful when the class identifies which of the following as the process of changing bilirubin from a fat-soluble product to a water-soluble product? A) Hemolysis B) Conjugation C) Jaundice D) Hyperbilirubinemia

Ans: B Feedback: The process in which bilirubin is changed from a fat-soluble product to a water-soluble product is called conjugation. Hemolysis involves the breakdown of blood cells. In the newborn, hemolysis of the red blood cells is the principal source of bilirubin. Jaundice is the manifestation of increased bilirubin in the bloodstream. Hyperbilirubinemia refers to the increased level of bilirubin in the blood.

27. Which of the following would be most appropriate for the nurse to suggest about pushing to a woman in the second stage of labor? A) "Lying flat with your head elevated on two pillows makes pushing easier." B) "Choose whatever method you feel most comfortable with for pushing." C) " Let me help you decide when it is time to start pushing." D) "Bear down like you're having a bowel movement with every contraction."

Ans: B Feedback: The role of the nurse should be to support the woman in her choice of pushing method and to encourage confidence in her maternal instinct of when and how to push. In the absence of any complications, nurses should not be controlling this stage of labor, but empowering women to achieve a satisfying experience. Common practice in many labor units is still to coach women to use closed glottis pushing with every contraction, starting at 10 cm of dilation, a practice that is not supported by research. Research suggests that directed pushing during the second stage may be accompanied by a significant decline in fetal pH and may cause maternal muscle and nerve damage if done too early. Effective pushing can be achieved by assisting the woman to assume a more upright or squatting position. Supporting spontaneous pushing and encouraging women to choose their own method of pushing should be accepted as best clinical practice.

11. The nurse places a warmed blanket on the scale when weighing a newborn. The nurse does so to minimize heat loss via which mechanism? A) Evaporation B) Conduction C) Convection D) Radiation

Ans: B Feedback: Using a warmed cloth diaper or blanket to cover any cold surface, such as a scale, that touches a newborn directly helps to prevent heat loss through conduction. Drying a newborn and promptly changing wet linens, clothes, or diapers help reduce heat loss via evaporation. Keeping the newborn out of a direct cool draft, working inside an isolette as much as possible, and minimizing the opening of portholes help prevent heat loss via convection. Keeping cribs and isolettes away from outside walls, cold windows, and air conditioners and using radiant warmers while transporting newborns and performing procedures will help reduce heat loss via radiation.

20. A nurse is reviewing the fetal heart rate pattern and observes abrupt decreases in FHR below the baseline, appearing as a U-shape. The nurse interprets these changes as reflecting which of the following? A) Early decelerations B) Variable decelerations C) Prolonged decelerations D) Late decelerations

Ans: B Feedback: Variable decelerations present as visually apparent abrupt decreases in FHR below baseline and have an unpredictable shape on the FHR baseline, possibly demonstrating no consistent relationship to uterine contractions. The shape of variable decelerations may be U, V, or W, or they may not resemble other patterns. Early decelerations are visually apparent, usually symmetrical and characterized by a gradual decrease in the FHR in which the nadir (lowest point) occurs at the peak of the contraction. They are thought to be a result of fetal head compression that results in a reflex vagal response with a resultant slowing of the FHR during uterine contractions. Late decelerations are visually apparent, usually symmetrical, transitory decreases in FHR that occur after the peak of the contraction. The FHR does not return to baseline levels until well after the contraction has ended. Delayed timing of the deceleration occurs, with the nadir of the uterine contraction. Late decelerations are associated with uteroplacental insufficiency. Prolonged decelerations are abrupt FHR declines of at least 15 bpm that last longer than 2 minutes but less than 10 minutes.

10. A woman telephones her health care provider and reports that her "water just broke." Which suggestion by the nurse would be most appropriate? A) "Call us back when you start having contractions." B) "Come to the clinic or emergency department for an evaluation." C) "Drink 3 to 4 glasses of water and lie down." D) "Come in as soon as you feel the urge to push."

Ans: B Feedback: When the amniotic sac ruptures, the barrier to infection is gone and there is the danger of cord prolapse if engagement has not occurred. Therefore, the nurse should suggest that the woman come in for an evaluation. Calling back when contractions start, drinking water, and lying down are inappropriate because of the increased risk for infection and cord prolapse. Telling the client to wait until she feels the urge to push is inappropriate because this occurs during the second stage of labor.

13. After describing continuous internal electronic fetal monitoring to a laboring woman and her partner, which of the following would indicate the need for additional teaching? A) "This type of monitoring is the most accurate method for our baby." B) "Unfortunately, I'm going to have to stay quite still in bed while it is in place." C) "This type of monitoring can only be used after my membranes rupture." D) "You'll be inserting a special electrode into my baby's scalp."

Ans: B Feedback: With continuous internal electronic monitoring, maternal position changes and movement do not interfere with the quality of the tracing. Continuous internal monitoring is considered the most accurate method, but it can be used only if certain criteria are met, such as rupture of membranes. A spiral electrode is inserted into the fetal presenting part, usually the head.

22. A group of nursing students are reviewing the changes in the newborn's lungs that must occur to maintain respiratory function. The students demonstrate understanding of this information when they identify which of the following as the first event? A) Expansion of the lungs B) Increased pulmonary blood flow C) Initiation of respiratory movement D) Redistribution of cardiac output

Ans: C Feedback: Before the newborn's lungs can maintain respiratory function, the following events must occur: respiratory movement must be initiated; lungs must expand, functional residual capacity must be established, pulmonary blood flow must increase, and cardiac output must be redistributed.

12. While performing a physical assessment of a newborn boy, the nurse notes diffuse edema of the soft tissues of his scalp that crosses suture lines. The nurse documents this finding as: A) Molding B) Microcephaly C) Caput succedaneum D) Cephalhematoma

Ans: C Feedback: Caput succedaneum is localized edema on the scalp, a poorly demarcated soft tissue swelling that crosses the suture lines. Molding refers to the elongated shape of the fetal head as it accommodates to the passage through the birth canal. Microcephaly refers to a head circumference that is 2 standard deviations below average or less than 10% of normal parameters for gestational age. Cephalhematoma is a localized effusion of blood beneath the periosteum of the skull.

14. When planning the care of a woman in the active phase of labor, the nurse would anticipate assessing the fetal heart rate at which interval? A) Every 2 to 4 hours B) Every 45 to 60 minutes C) Every 15 to 30 minutes D) Every 10 to 15 minutes

Ans: C Feedback: During the active phase of labor, FHR is monitored every 15 to 30 minutes. FHR is assessed every 30 to 60 minutes during the latent phase of labor. The woman's temperature is typically assessed every 4 hours during the first stage of labor and every 2 hours after ruptured membranes. Blood pressure, pulse, and respirations are assessed every hour during the latent phase and every 30 minutes during the active and transition phases. Contractions are assessed every 30 to 60 minutes during the latent phase and every 15 to 30 minutes during the active phase, and every 15 minutes during transition.

16. The nurse is assessing the skin of a newborn and notes a rash on the newborn's face, and chest. The rash consists of small papules and is scattered with no pattern. The nurse interprets this finding as which of the following? A) Harlequin sign B) Nevus flammeus C) Erythema toxicum D) Port wine stain

Ans: C Feedback: Erythema toxicum (newborn rash) is a benign, idiopathic, generalized, transient rash that occurs in up to 70% of all newborns during the first week of life. It consists of small papules or pustules on the skin resembling flea bites. The rash is common on the face, chest, and back. One of the chief characteristics of this rash is its lack of pattern. It is caused by the newborn's eosinophils reacting to the environment as the immune system matures. Harlequin sign refers to the dilation of blood vessels on only one side of the body, giving the newborn the appearance of wearing a clown suit. It gives a distinct midline demarcation, which is described as pale on the nondependent side and red on the opposite, dependent side. Nevus flammeus or port wine stain is a capillary angioma located directly below the dermis. It is flat with sharp demarcations and is purple-red. This skin lesion is made up of mature capillaries that are congested and dilated.

1. A woman in her 40th week of pregnancy calls the nurse at the clinic and says she's not sure whether she is in true or false labor. Which statement by the client would lead the nurse to suspect that the woman is experiencing false labor? A) "I'm feeling contractions mostly in my back." B) "My contractions are about 6 minutes apart and regular." C) "The contractions slow down when I walk around." D) "If I try to talk to my partner during a contraction, I can't."

Ans: C Feedback: False labor is characterized by contractions that are irregular and weak, often slowing down with walking or a position change. True labor contractions begin in the back and radiate around toward the front of the abdomen. They are regular and become stronger over time; the woman may find it extremely difficult if not I'm to have a conversation during a contraction.

9. When assessing fetal heart rate, the nurse finds a heart rate of 175 bpm, accompanied by a decrease in variability and late decelerations. Which of the following would the nurse do next? A) Have the woman change her position. B) Administer oxygen. C) Notify the health care provider. D) Continue to monitor the pattern every 15 minutes.

Ans: C Feedback: Fetal tachycardia as evidenced by a fetal heart rate greater than 160 bpm accompanied by a decrease in variability and late decelerations is an ominous sign indicating the need for prompt intervention. The health care provider should be notified immediately and then measures should be instituted such as having the woman lie on her side and administering oxygen. In this instance, monitoring should be continuous to detect any further changes and evaluate the effectiveness of interventions.

9. The nurse completes the initial assessment of a newborn. Which finding would lead the nurse to suspect that the newborn is experiencing difficulty with oxygenation? A) Respiratory rate of 54 breaths/minute B) Abdominal breathing C) Nasal flaring D) Acrocyanosis

Ans: C Feedback: Nasal flaring is a sign of respiratory difficulty in the newborn. A rate of 54 breaths/minute, diaphragmatic/abdominal breathing, and acrocyanosis are normal findings.

30. A nurse is providing care to a woman during the third stage of labor. Which of the following would alert the nurse that the placenta is separating? (Select all that apply.) A) Boggy, soft uterus B) Uterus becoming discoid shaped C) Sudden gush of dark blood from the vagina D) Shortening of the umbilical cord

Ans: C Feedback: Signs that the placenta is separating include a firmly contracting uterus, a change in uterine shape from discoid to globular ovoid, a sudden gush of dark blood from the vaginal opening, and lengthening of the umbilical cord protruding from the vagina.

27. While observing the interaction between a newborn and his mother, the nurse notes the newborn nestling into the arms of his mother. The nurse identifies this behavior as which of the following? A) Habituation B) Self-quieting ability C) Social behaviors D) Orientation

Ans: C Feedback: Social behaviors include cuddling and snuggling into the arms of the parent when the newborn is held. Habituation self-quieting ability refers to newborns' ability to quiet and comfort themselves, such as by hand-to-mouth movements and sucking, alerting to external stimuli and motor activity. Habituation is the newborn's ability to process and respond to visual and auditory stimuli—that is, how well and appropriately he or she responds to the environment. Habituation is the ability to block out external stimuli after the newborn has become accustomed to the activity. Orientation refers to the response of newborns to stimuli, becoming more alert when sensing a new stimulus in their environment.

26. Assessment of a pregnant woman reveals that the presenting part of the fetus is at the level of the maternal ischial spines. The nurse documents this as which station? A) -2 B) -1 C) 0 D) +1 .

Ans: C Feedback: Station refers to the relationship of the presenting part to the level of the maternal pelvic ischial spines. Fetal station is measured in centimeters and is referred to as a minus or plus, depending on its location above or below the ischial spines. Zero (0) station is designated when the presenting part is at the level of the maternal ischial spines. When the presenting part is above the ischial spines, the distance is recorded as minus stations. When the presenting part is below the ischial spines, the distance is recorded as plus stations

5. A woman is admitted to the labor and birthing suite. Vaginal examination reveals that the presenting part is approximately 2 cm above the ischial spines. The nurse documents this finding as: A) +2 station B) 0 station C) -2 station D) Crowning

Ans: C Feedback: The ischial spines serve as landmarks and are designated as zero status. If the presenting part is palpated higher than the maternal ischial spines, a negative number is assigned. Therefore, the nurse would document the finding as -2 station. If the presenting part is below the ischial spines, then the station would be +2. Crowning refers to the appearance of the fetal head at the vaginal opening.

17. The nurse is reviewing the medical record of a woman in labor and notes that the fetal position is documented as LSA. The nurse interprets this information as indicating which of the following is the presenting part? A) Occiput B) Face C) Buttocks D) Shoulder

Ans: C Feedback: The second letter denotes the presenting part which in this case is "S" or the sacrum or buttocks. The letter "O" would denote the occiput or vertex presentation. The letter "M" would denote the mentum (chin) or face presentation. The letter "A" would denote the acromion or shoulder presentation.

6. A client has not received any medication during her labor. She is having frequent contractions every 1 to 2 minutes and has become irritable with her coach and no longer will allow the nurse to palpate her fundus during contractions. Her cervix is 8 cm dilated and 90% effaced. The nurse interprets these findings as indicating: A) Latent phase of the first stage of labor B) Active phase of the first stage of labor C) Transition phase of the first stage of labor D) Pelvic phase of the second stage of labor

Ans: C Feedback: The transition phase is characterized by cervical dilation of 8 to 10 cm, effacement of 80% to 100%, contractions that are strong, painful, and frequent (every 1 to 2 minutes) and last 60 to 90 seconds, and irritability, apprehension, and feelings of loss of control. The latent phase is characterized by mild contractions every 5 to 10 minutes, cervical dilation of 0 to 3 cm and effacement of 0% to 40%, and excitement and frequent talking by the mother. The active phase is characterized by moderate to strong contractions every 2 to 5 minutes, cervical dilation of 4 to 7 cm and effacement of 40% to 80%, with the mother becoming intense and inwardly focused. The pelvic phase of the second stage of labor is characterized by complete cervical dilation and effacement, with strong contractions every 2 to 3 minutes; the mother focuses on pushing.

8. The nurse places a newborn with jaundice under the phototherapy lights in the nursery to achieve which goal? A) Prevent cold stress B) Increase surfactant levels in the lungs C) Promote respiratory stability D) Decrease the serum bilirubin level

Ans: D Feedback: Jaundice reflects elevated serum bilirubin levels; phototherapy helps to break down the bilirubin for excretion. Phototherapy has no effect on body temperature, surfactant levels, or respiratory stability.

5. The nurse institutes measures to maintain thermoregulation based on the understanding that newborns have limited ability to regulate body temperature because they: A) Have a smaller body surface compared to body mass B) Lose more body heat when they sweat than adults C) Have an abundant amount of subcutaneous fat all over D) Are unable to shiver effectively to increase heat production

Ans: D Feedback: Newborns have difficulty maintaining their body heat through shivering and other mechanisms. They have a large body surface area relative to body weight and have limited sweating ability. Additionally, newborns lack subcutaneous fat to provide insulation.

22. A group of nursing students are reviewing the various medications used for pain relief during labor. The students demonstrate understanding of the information when they identify which agent as the most commonly used opioid? A) Butorphanol B) Nalbuphine C) Fentanyl D) Meperidine

Ans: D Feedback: Of all of the synthetic opioids (butorphanol [Stadol], nalbuphine [Nubain], fentanyl [Sublimaze], and meperidine [Demerol]), meperidine is the most commonly used opioid for the management of pain during labor.

20. A nurse is assessing a newborn and observes the newborn moving his head and eyes toward a loud sound. The nurse interprets this as which of the following? A) Habituation B) Motor maturity C) Social behavior D) Orientation

Ans: D Feedback: Orientation refers to the response of newborns to stimuli. It reflects newborns' response to auditory and visual stimuli, demonstrated by their movement of head and eyes to focus on that stimulus. Habituation is the newborn's ability to process and respond to visual and auditory stimuli—that is, how well and appropriately he or she responds to the environment. Habituation is the ability to block out external stimuli after the newborn has become accustomed to the activity. Motor maturity depends on gestational age and involves evaluation of posture, tone, coordination, and movements. These activities enable newborns to control and coordinate movement. When stimulated, newborns with good motor organization demonstrate movements that are rhythmic and spontaneous. Social behaviors include cuddling and snuggling into the arms of the parent when the newborn is held.

12. Which of the following would alert the nurse to the possibility of respiratory distress in a newborn? A) Symmetrical chest movements B) Periodic breathing C) Respirations of 40 breaths/minute D) Sternal retractions

Ans: D Feedback: Sternal retractions, cyanosis, tachypnea, expiratory grunting, and nasal flaring are signs of respiratory distress in a newborn. Symmetrical chest movements and a respiratory rate between 30 to 60 breaths/minute are typical newborn findings. Some newborns may demonstrate periodic breathing (cessation of breathing lasting 5 to 10 seconds without changes in color or heart rate) in the first few days of life.

2. A new mother reports that her newborn often spits up after feeding. Assessment reveals regurgitation. The nurse responds, integrating understanding that this most likely is due to which of the following? A) Placing the newborn prone after feeding B) Limited ability of digestive enzymes C) Underdeveloped pyloric sphincter D) Relaxed cardiac sphincter

Ans: D Feedback: The cardiac sphincter and nervous control of the stomach is immature, which may lead to uncoordinated peristaltic activity and frequent regurgitation. Placement of the newborn is unrelated to regurgitation. Most digestive enzymes are available at birth, but they are limited in their ability to digest complex carbohydrates and fats; this results in fatty stools, not regurgitation. Immaturity of the pharyngoesophageal sphincter and absence of lower esophageal peristaltic waves, not an underdeveloped pyloric sphincter, also contribute to the reflux of gastric contents.

11. While making rounds in the nursery, the nurse sees a 6-hour-old baby girl gagging and turning bluish. Which of the following would the nurse do first? A) Alert the physician stat and turn the newborn to her right side. B) Administer oxygen via facial mask by positive pressure. C) Lower the newborn's head to stimulate crying. D) Aspirate the oral and nasal pharynx with a bulb syringe.

Ans: D Feedback: The nurse's first action would be to suction the oral and nasal pharynx with a bulb syringe to maintain airway patency. Turning the newborn to her right side will not alleviate the blockage due to secretions. Administering oxygen via positive pressure is not indicated at this time. Lowering the newborn's head would be inappropriate.

15. The nurse administers vitamin K intramuscularly to the newborn based on which of the following rationales? A) Stop Rh sensitization B) Increase erythropoiesis C) Enhance bilirubin breakdown D) Promote blood clotting

Ans: D Feedback: Vitamin K promotes blood clotting by increasing the synthesis of prothrombin by the liver. RhoGAM prevents Rh sensitization. Erythropoietin stimulates erythropoiesis. Phototherapy enhances bilirubin breakdown.

12. A fetus is assessed at 2 cm above the ischial spines. The nurse would document fetal station as: A) +4 B) +2 C) 0 D) -2

Ans: D Feedback: When the presenting part is above the ischial spines, it is noted as a negative station. Since the measurement is 2 cm, the station would be -2. A 0 station indicates that the fetal presenting part is at the level of the ischial spines. Positive stations indicate that the presenting part is below the level of the ischial spines.


Ensembles d'études connexes

History Chapter 22 Study Guide Section 1

View Set

Clinical Chem/Urinanalysis Midterm

View Set